Sunteți pe pagina 1din 65

1 One of three blood culture bottles drawn from a patient with unexplained fevers reveals gram-positive cocci growing

in clusters. Which of the following tests would be most useful in determining whether this organism is a part of the normal skin flora? A. Bacitracin resistance B. Catalase C. Coagulase D. Novobiocin resistance E. Optochin resistance Explanation: The correct answer is C. Gram-positive cocci in clusters are staphylococci. Staphylococcus aureus is a common pathogen that should not be considered normal skin flora. Other staphylococci, especially Staphylococcus epidermidis, may contaminate blood cultures and can be differentiated from Staphylococcus aureus by the coagulase test. S. aureus is the only coagulase-positive staphylococcus. Bacitracin sensitivity differentiates Streptococcus pyogenes from the other beta-hemolytic streptococci, which are bacitracin resistant (choice A). The catalase test (choice B) is used to differentiate streptococci from staphylococci. Staphylococci produce catalase and can generate oxygen bubbles in hydrogen peroxide, whereas streptococci cannot. Novobiocin resistance (choice D) differentiates the coagulase-negative staphylococci into S. epidermidis (novobiocin sensitive) and S. saprophyticus (novobiocin resistant). Optochin resistance (choice E) differentiates the major pathogenic alpha-hemolytic streptococci. S. pneumoniae is optochin- and bile-sensitive whereas S. viridans is resistant to both optochin and bile. A 4-month-old infant presents with failure to thrive, progressive muscular weakness, and poor head control. On questioning, the mother states that she typically feeds the baby soy-based formula sweetened with honey. Which of the following organisms is most likely to be responsible for the child's presentation? A. Clostridium botulinum B. Clostridium difficile C. Clostridium perfringens D. Clostridium tetani E. Corynebacterium diphtheriae Explanation: The correct answer is A. The baby has infant botulism (floppy baby syndrome), which is due to germination of Clostridium botulinum spores (found in honey) in the baby's gastrointestinal tract. Patients improve when honey is removed from the diet. This disorder is most common in children under the age of six months; older children and adults do not appear to be vulnerable to this form of botulism, but are susceptible to botulism caused by ingestion of preformed toxin. Clostridium difficile(choice B) causes pseudomembranous colitis, especially after antibiotic therapy.

2 Clostridium perfringens(choice C) causes gas gangrene and gastroenteritis, and it is not associated with ingestion of honey. Clostridium tetani (choice D) causes tetanus, and does not cause a food-borne illness in infants. Corynebacterium diphtheriae(choice E) causes diphtheria in susceptible individuals.

A 35-year-old woman presents to her gynecologist with complaints of burning on urination for the past 2 days. Dipstick test of her urine demonstrates marked positivity for leukocyte esterase, but no reactivity for nitrite. Urine culture later grows out large numbers of organisms. Which of the following bacteria are most likely to be responsible for this patient's infection? A. Enterobacter sp. B. Enterococcus faecalis C. Escherichia coli D. Klebsiella pneumoniae E. Pseudomonas aeruginosa Explanation: The correct answer is B. The positive leukocyte esterase test indicates the presence of neutrophils in the urine, suggesting a bacterial infection. The nitrite test exploits the fact that most Enterobacteria (gram-negative enteric rods) are able to form nitrite from nitrate; thus, the nitrite test is used to diagnose urinary tract infections. One limitation of this method is the fact that enterococci (gut streptococci) do not produce nitrite from nitrate, but can nonetheless cause urinary tract infections. Enterococcal urinary tract infections are often nosocomial and classically acquired in the intensive care unit, although they can occur in other settings. Enterobacter sp. (choice A), Escherichia coli(choice C), Klebsiella pneumoniae(choice D), and Pseudomonas aeruginosa(choice E) can cause urinary tract infections and would usually be picked up by the dipstick for nitrites. False-negative results might still be seen with these organisms if the infection was light, the bladder had been recently emptied prior to collection, and the urine was "new" and had not yet grown enough bacteria to produce a positive result.

A 37-year-old woman presents with 3 days of progressive joint pain in her ankles, knees, and wrists. She recalls three similar episodes over the past several years. On examination, she has a temperature of 38.7 C, her blood pressure is 110/70 mm Hg, and her heart rate is 90/min. She has a diffuse petechial rash over her trunk and extensor surfaces. Her ankles and knees are swollen, red, and tender with decreased range of motion, and there is tenderness over the tendon sheaths of her hands and forearms. Blood cultures are negative. Aspiration of joint fluid reveals a white cell count of 22,000/mm3 with no visible organisms, but culture on chocolate agar is positive. Which of the following is an attribute of the causative organism that allows it to produce recurrent infections? A. It is an intracellular pathogen B. It is resistant to ceftriaxone C. It is resistant to complement-mediated lysis D. Its capsule is not immunogenic

3 E. Its pili undergo antigenic and phase variation Explanation: The correct answer is E. This is a case of Neisseria gonorrhoeae arthritis. Patients are continuously susceptible to reinfection because of antigenic variation and phase variation of the pili. N. gonorrhoeae is not an intracellular pathogen (choice A), although it may be found intracellularly in neutrophils after it has been phagocytized. Ceftriaxone (choice B) is the drug of choice for N. gonorrhoeae. Gonococci are especially susceptible to complement-mediated lysis, not resistant to it (choice C). N. gonorrhoeae has an insignificant capsule that does not play a major role in the pathogenesis, but its capsule is immunogenic (compare with choice D). Streptococcus pyogenes is the best known example of a nonimmunogenic capsule, made of hyaluronic acid, but virtually all other capsules are immunogenic.

A Pap smear from a woman with chronic cervicitis shows cytoplasmic inclusions within epithelial cells. Fluorescent antibodies identify both these inclusions and "elementary bodies." The intracellular organisms causing the infection are unusual because they cannot synthesize which of the following? A. ATP B. Cholesterol C. DNA D. Proteins E. RNA Explanation: The correct answer is A. The disease is chlamydial cervicitis. This venereally transmitted infection is usually suspected after treatment for gonorrhea fails to relieve symptoms. However, a few cases are picked up when cytoplasmic inclusions composed of aggregates of chlamydia are identified on Pap smears. Confirmation can be made with fluorescent antibodies that pick up both the aggregates and individual bacteria known as "elementary bodies." The organisms are obligate intracellular parasites because they have lost the ability to synthesize ATP. Although some authors consider them to be "bacteria," others do not, since this is such a fundamental difference between the chlamydia and free living bacteria. Failure to synthesize cholesterol (choice B) is not usually cited as a problem specific to any type of organism, although viruses are not able to do so without using host machinery. Some viruses cannot synthesize DNA (choice C), but the disease in question is not caused by a virus. Protein synthesis (choice D) by viruses requires host ribosomes. Some viruses cannot synthesize RNA (choice E), but the disease in question is not caused by a virus.

A patient with colorectal cancer develops septicemia complicated by endocarditis. You would expect the blood cultures to grow

4 A. Streptococcus agalactiae B. Streptococcus bovis C. Streptococcus pneumoniae D. Streptococcus pyogenes E. Streptococcus viridans Explanation: The correct answer is B.Streptococcus bovis is a Group D streptococcus. There is a significant association between S. bovis bacteremia and endocarditis with carcinoma of the colon and other colonic diseases. Every patient with S. bovis bacteremia should undergo gastrointestinal and cardiac evaluation. Up to 50% of patients with S. bovis bacteremia are reported to have underlying colonic malignancies. In another study, 25-50% of cases of S. bovis bacteremia were associated with endocarditis, especially in patients with preexisting valvular lesions. Streptococcus agalactiae(choice A) is an important cause of maternal and neonatal bacteremia and neonatal meningitis. It is part of the normal flora of the gastrointestinal tract and the female genital tract. Streptococcus pneumoniae(choice C) is a leading cause of community-acquired pneumonia, meningitis in adults, otitis media (especially in children), and sinusitis. Spontaneous peritonitis due to S. pneumoniae is reported in children with ascites from nephrotic syndrome. Asplenia predisposes patients to severe infections with S. pneumoniae and other encapsulated organisms. S. pneumoniae infections are also more frequent and unusually severe in patients with sickle cell anemia, multiple myeloma, alcoholism, and hypogammaglobulinemia. S. pneumoniae is now the leading cause of invasive bacterial respiratory disease in patients with AIDS. Streptococcus pyogenes(choice D) is the most common cause of bacterial pharyngitis. Complications include paratonsillar abscesses, otitis media, and sinusitis. Long-term sequelae include rheumatic fever and poststreptococcal glomerulonephritis. S. pyogenes is also responsible for many skin and soft tissue infections. The organism also produces many toxins that produce a variety of diseases. Streptococcus viridans(choice E), or the viridans Streptococci, are the most common cause of subacute bacterial endocarditis, which should be suspected in cases of viridans streptococcal bacteremia. One species of viridans Streptococci, Streptococcus milleri, is frequently associated with pyogenic abscesses, especially of the liver.

A 54-year-old woman suffering from influenza deteriorates and develops shaking chills and a high fever. Physical examination is remarkable for dullness to percussion at the left base and decreased breath sounds on the left. Chest x-ray confirms the diagnosis of lobar pneumonia, presumed to be caused by Streptococcus pneumoniae. The patient has no known drug allergies. Which of the following antibiotics would be most appropriate to treat the patient's condition? A. Cefotaxime B. Chloramphenicol C. Erythromycin D. Penicillin E. Vancomycin

5 Explanation: The correct answer is D. Penicillin remains the first-line drug of choice for pneumococcal pneumonia, except in patients with penicillin allergy and in the relatively few areas in which pneumococcal strains with high-level penicillin resistance exist. Alternative therapies include erythromycin and vancomycin. The third-generation cephalosporin cefotaxime (choice A) is not usually used for pneumococcal pneumonia. Chloramphenicol (choice B) is not usually used for pneumococcal pneumonia. Erythromycin (choice C) is a good alternative therapy for pneumococcal pneumonia, but is usually used only when a penicillin allergy is present. Vancomycin (choice E) is not the first-line therapy, but it is a good alternative in patients allergic to penicillin or when high-level penicillin resistance (relatively uncommon) is present.

An otherwise healthy 3-year-old child is brought to the pediatrician with umbilicated, flesh-colored papules on his trunk. This condition is related to infection with which of the following viruses? A. Cytomegalovirus B. Herpesvirus 6 C. Parvovirus D. Poxvirus E. Variola Explanation: The correct answer is D. The lesions are characteristic of molluscum contagiosum, which is a typically benign and self-limited condition caused by a poxvirus. The disease can be transmitted either venereally or through non-venereal contact. The other viruses listed do not cause similar skin lesions. Patients with advanced HIV infection may develop a severe, generalized, and persistent eruption, often involving the face and upper body. Cytomegalovirus (choice A) causes congenital infections and disseminated infections in immunosuppressed patients. Herpesvirus 6 (choice B)causes roseola (exanthem subitum). Parvovirus (choice C) causes aplastic crises in patients with hemolytic anemia. Variola (choice E) is the smallpox virus.

A 39-year-old black man presents with complaints of anorexia, malaise, fatigue, dark urine, and upper abdominal discomfort. He admits to homosexuality, but denies blood transfusions, alcohol intake, or intravenous drug abuse. On physical examination, the patient has a temperature of 100.2 degrees F, scleral icterus, and jaundice. His liver is palpable below the right costal margin, and there is moderate right upper quadrant tenderness. Liver function test results are as follows: total bilirubin 12.4%, SGOT 980 units, SGPT 1200 units. Serologic findings are as follows: anti-hepatitis A IgM negative, HBsAg positive, anti-HBc IgM positive, HBeAg positive, anti-HBsAb negative, hepatitis C negative. When this patient enters the window period, what would likely be the first change in his serologic findings?

6 A. He will become HBcAg positive B. He will become HBc IgG positive C. He will become HBeAg negative D. He will become HBsAb positive E. He will become HBsAg negative Explanation: The correct answer is E. This patient has acute hepatitis B. The "window period" refers to that period in infection when neither hepatitis B surface antigen (HBsAg) nor its antibody (HBsAb) can be detected in the serum of the patient. It is an immunologically mediated phenomenon caused by the precipitation of antigen-antibody complexes in their zone of equivalent concentrations and, thereby, their removal from the circulation. Because of this, the first thing that will happen in the window period is that the serum will become negative for the surface antigen (HBsAg), as that antigen is precipitated out of the serum by developing levels of its specific antibody (HBsAb). HBcAg is not typically measured (choice A) in the serum. Levels of the c-core antibody HBc IgG (choice B) and HBeAg (choice C) do not have a relationship to the window period. Levels of HBsAb (choice D) will not be detectable until there is antibody excess, and the patient is leaving the window period.

A 15-year-old girl in a rural community has swollen, painful lymph nodes in her right axilla. Physical examination reveals multiple scratches on her right arm with a papule associated with one of the scratch marks. She states that the scratches occurred about 5 days ago. What type of animal is the most likely source of the infection? A. Cat or kitten B. Chicken C. Dog or puppy D. Horse E. Parrot Explanation: The correct answer is A. This patient has the classic symptoms of cat scratch disease caused by the bacillus Bartonella henselae. The disease is self-limited with the onset of symptoms occurring 3-10 days following an inoculating scratch. The organism can be isolated from kittens, typically less than 1 year of age, or from fleas. A history of a new kitten in the house and the papule at the site of a scratch with regional painful adenopathy defines the classic scenario. Chickens (choice B) can harbor Salmonella spp. producing a gastroenteritis or enterocolitis. Chicken guano is also a favorable environment for the fungus Histoplasma capsulatum. The mycelial phase thrives in the rich soil. The human disease is a granulomatous infection involving the lungs and mimicking tuberculosis. Dogs or puppies (choice C) carry Capnocytophaga canimorsus as part of the normal flora of the oral cavity.

7 Infections from licking or biting range from a selflimited cellulitis to fatal septicemia. Patients at risk for more severe infections are those with asplenia, alcoholism, or hematologic malignancies. This organism is also associated with cat bites, but the patient develops cellulitis and fulminant septicemia, especially in asplenic patients. Pasteurella multocida is another pathogen that colonizes the nasopharynx and gastrointestinal tract of cats and dogs. Cats have the highest rate of colonization (50-90%), followed by dogs (50%), swine (50%), and rats (14%). P. multocida most commonly causes a localized soft tissue infection or cellulitis after an animal bite, but systemic symptoms may be present in about 40% of the cases. These symptoms include osteomyelitis, septic arthritis, or tenosynovitis. Horses (choice D) and horse manure have been associated with a pulmonary opportunistic infection with cavitation caused by Rhodococcus equi that resembles tuberculosis in immunocompromised patients. Burkholderia mallei (the cause of glanders) is characterized by non-caseating granulomatous abscesses of skin, lymphadenopathy, and pronounced involvement of the lungs. Parrots (choice E) are associated with psittacosis caused by Chlamydia psittaci. Psittacosis is associated with a dry, hacking cough productive of scant sputum, an interstitial infiltrate in the lungs, severe headache, and myalgias. A pale macular rash is also seen.

A 73-year-old woman with a history of diabetes presents with left ear pain and drainage of pus from the ear canal. She has swelling and tenderness over the left mastoid bone. Which of the following microorganisms is the most likely causative agent? A. Hemophilus influenzae B. Klebsiella pneumoniae C. Mucor sp. D. Pseudomonas aeruginosa E. Streptococcus pyogenes Explanation: The correct answer is D.Pseudomonas aeruginosa causes malignant otitis externa, which is a severe necrotizing infection of the external ear canal. Infection tends to spread to the mastoid bone, temporal bone, sigmoid sinus, base of the skull, meninges, and brain. Patients at increased risk include the elderly, those with diabetes, and the immunocompromised. Pseudomonas is also associated with many other clinical syndromes, including infection following traumatic wounds to the feet in persons wearing sneakers or rubber-soled shoes, since the organism can be cultured from the shoes. Hemophilus influenzae(choice A) produces a variety of clinical syndromes. H.Influenzae is the third-most common cause of meningitis in children aged 1 month to 18 years. It is the most common cause of acute epiglottitis, the most common cause of purulent bacterial conjunctivitis, and the second-most common cause of otitis media. Patients at risk include those with COPD and cystic fibrosis, alcoholics, splenectomized patients, and young patients. Klebsiella pneumoniae(choice B) is a gram-negative organism that produces a necrotizing pneumonia in diabetics and alcoholics. Patients typically present with an abrupt onset of fever, shaking chills, and purulent, foul-smelling sputum. Mucor (choice C) is a fungal infection that is particularly severe in the diabetic or the immunocompromised patient. In the acidotic diabetic, the fungus produces a life-threatening, invasive rhinocerebral infection. The infection begins in the nasal passages, extends into the paranasal sinuses, and spreads through the cribiform plate to the frontal lobes of the brain. Patients typically complain of headache, facial pain, and orbital swelling. Streptococcus pyogenes(choice E) causes bacterial pharyngitis, otitis media, and sinusitis. It is also associated

8 with toxin-related diseases and skin infections.

A 29-year-old female diagnosed with AIDS has been suffering from a progressive blurring of vision in her right eye. On funduscopic examination, a small white opaque lesion is noted on the retina of her right eye. Which of the following is the most appropriate therapy for this patient? A. Acyclovir B. Amantadine C. Flucytosine D. Ganciclovir E. Zidovudine Explanation: The correct choice is D. This patient is most likely suffering from cytomegalovirus (CMV) retinitis. The best drug treatment for this infection is ganciclovir. Acyclovir (choice A) is not effective in CMV infections. It is used more for HSV type 1 and 2 infections. Amantadine (choice B) is used either therapeutically or prophylactically for the influenza A virus. Flucytosine (choice C) is an antifungal agent. Zidovudine (choice E) is a first-line drug for the treatment of AIDS. The drug by itself is ineffective against CMV retinitis. A newborn is infected in utero with an enveloped virus containing double-stranded DNA. The child develops petechiae, hepatosplenomegaly, and jaundice. Brain calcifications are detected on computed tomography (CT). With which of the following viruses is the newborn most likely infected? A. Cytomegalovirus B. Hepatitis B C. Hepatitis C D. Hepatitis D E. Herpes simplex Explanation: The correct answer is A. The most common agents causing congenital infection in the United States are cytomegalovirus, herpes simplex virus, and Treponema pallidum. Congenital cytomegalovirus infection, which occurs in 1 to 2% of all live births in the U.S., results from transplacental acquisition of a primary or recurrent maternal infection. Most infections are asymptomatic, but about 5% will present with hepatosplenomegaly, periventricular brain calcification, petechial hemorrhages, and hydrops. Sensorineural deafness is a common manifestation in affected infants. Primary infection of the mother is strongly associated with more severe symptoms in affected infants. The mortality rate is 30% and most of the survivors will have neurologic impairment. CMV infections in children and adults are often asymptomatic; however, the virus is responsible for

9 10% of infectious mononucleosis cases (characterized by an absence of sheep RBC agglutinins). CMV also causes serious disease in immunosuppressed patients, including bone marrow recipients and AIDS patients. Ganciclovir is the drug of choice, although foscarnet has also been used with some success. Infections with hepatitis viruses (choices B, C, and D), if symptomatic, are characterized by jaundice, lethargy, failure to thrive, abdominal distention, clay- colored stools, and elevated transaminase levels. Hepatitis B(choice B) is the only viral hepatitis agent that is recognized as an important cause of neonatal disease. The infection is usually acquired during the birth process; therapy includes hyperimmune human anti-hepatitis B serum followed a few weeks later with active immunization with rHBsAg injections. Herpes simplex (choice E) can also infect the neonate. The hallmark of infection is painful skin vesicles (present in 50% of affected infants); other manifestations include encephalitis, pneumonia, hepatitis, and disseminated intravascular coagulopathy. Massive hepatic and adrenal necrosis is common. Therapy is with acyclovir.

A 60-year-old alcoholic smoker abruptly develops high fever, shakes, a severe headache, and muscle pain. He initially has a dry, insignificant cough, but over the next few days he develops marked shortness of breath requiring assisted ventilation. Chest x-ray demonstrates homogeneous radiographic shadowing that initially involves the left lower lobe but continues to spread until both lungs are extensively involved. Culture of bronchoalveolar lavage fluid on buffered charcoal yeast extract (BCYE) demonstrates a coccobacillary pathogen. Which of the following is the most likely causative organism? A. Legionella pneumophila B. Listeria monocytogenes C. Spirillium minus D. Staphylococcus aureus E. Streptococcus pneumoniae Explanation: The correct answer is A. The patient has a severe, potentially fatal, pneumonia with prominent systemic symptoms. Culture on BCYE is the specific clue that the organism is Legionella pneumophila. The disease is respiratory Legionellosis, also known as Legionnaire's disease, because the disease was first described when it occurred in epidemic form following an American Legion convention at a Philadelphia hotel. Patients tend to be older (40-70 years old) and may have risk factors including cigarette use, alcoholism, diabetes, chronic illness, or immunosuppressive therapy. Listeria monocytogenes(choice B) causes listeriosis and is not a notable cause of pneumonia. Spirillium minus (choice C) is a cause of rat-bite fever and is not a notable cause of pneumonia. Staphylococcus aureus(choice D) can cause pneumonia, but is easily cultured on routine media. Streptococcus pneumoniae(choice E) can cause pneumonia, but is easily cultured on routine media.

A 33-year-old woman presents with fever, vomiting, severe irritative voiding symptoms, and pronounced costovertebral angle tenderness. Laboratory evaluation reveals leukocytosis with a left shift; blood cultures indicate bacteremia. Urinalysis shows pyuria, mild hematuria, and gram-negative bacteria. Which of the following drugs would best treat this patient's infection?

10 A. Ampicillin and gentamicin B. Erythromycin C. Gentamicin and vancomycin D. Phenazopyridine and nitrofurantoin E. Tetracycline Explanation: The correct answer is A. Acute pyelonephritis is an infectious disease involving the kidney parenchyma and the renal pelvis. Gram-negative bacteria, such as Escherichia coli, Proteus, Klebsiella, and Enterobacter, are the most common causative organisms in acute pyelonephritis. Laboratory evaluation will often reveal leukocytosis with a left shift, and urinalysis typically shows pyuria, varying degrees of hematuria, and white cell casts. Since bacteremia is present, the patient should be hospitalized and empirically started on IV ampicillin and gentamicin. This regimen may be need to be changed, however, once the sensitivity results are available. Erythromycin (choice B) and tetracycline (choice E) are both bacteriostatic antibiotics and would not be recommended in a patient with a severe infection, such as acute pyelonephritis with bacteremia. Vancomycin (choice C) is primarily used in the treatment of severe gram-positive infections. Phenazopyridine (choice D) is a urinary analgesic, and nitrofurantoin (choice D) is a urinary tract anti-infective. Although nitrofurantoin is indicated for the treatment of "mild" cases of pyelonephritis, as well as cystitis, this patient's condition is severe and should be treated with appropriate antibiotics.

A newborn infant has multiple, hemorrhagic, cutaneous lesions and does not respond to sound. Head CT scan shows periventricular calcifications. Which of the following infectious agents is the most likely cause of this child's presentation? A. Cytomegalovirus B. Herpes simplex C. Rubella D. Syphilis E. Toxoplasmosis Explanation: The correct answer is A. The infectious agents listed are all important causes of congenital disease. The triad of cutaneous hemorrhages ("blueberry muffin baby"), deafness, and periventricular CNS calcifications suggests congenital CMV infection, the most common cause of intrauterine fetal viral infection. Other manifestations include microcephaly and hepatosplenomegaly. Neonatal herpes (choice B) may be congenital, but more commonly is acquired during vaginal delivery. The infection is characterized by vesicles on the skin and mucous membranes, encephalitis, or disseminated disease. Congenital rubella (choice C) can cause mental retardation, heart abnormalities, blindness, encephalitis, and motor abnormalities.

11 Congenital syphilis (choice D) can cause death in utero, bones, and central nervous system. or a variety of problems including abnormal teeth,

Toxoplasmosis (choice E) can be either acquired during delivery (mild) or congenital (severe). Severe infections can cause stillbirth, chorioretinitis, intracerebral calcifications, and hydro- or microcephaly.

A very ill neonate has widespread granulomas. In utero infection with which of the following organisms is suggested by this finding? A. Clostridium botulinum B. Escherichia coli C. Haemophilus influenzae D. Listeria monocytogenes E. Neisseria gonorrhoeae Explanation: The correct answer is D. All of the organisms listed can cause infection in infants, but Listeria monocytogenes is the one to cause potentially fatal granulomas (granulomatis infantiseptica) following in utero infection. Clostridium botulinum(choice A) can cause infant botulism, characterized by failure to thrive, and progressive muscular weakness. Escherichia coli(choice B) can cause diarrhea, pneumonia, and meningitis in infants. Haemophilus influenzae(choice C) can cause acute epiglottitis, meningitis, pneumonia, and otitis media in young children. Neisseria gonorrhoeae(choice E) can be transmitted via the birth canal, and causes ophthalmia neonatorum. A 37-year-old female presents to the emergency room with a fever. Chest x-ray shows multiple patchy infiltrates in both lungs. Echocardiography and blood cultures suggest a diagnosis of acute bacterial endocarditis limited to the tricuspid valve. Which of the following is the most probable etiology? A. Congenital heart disease B. Illicit drug use C. Rheumatic fever D. Rheumatoid arthritis E. Systemic lupus erythematosus Explanation: The correct answer is B. The most probable etiology of bacterial endocarditis involving the tricuspid valve is illicit intravenous drug use, which can introduce skin organisms into the venous system that then attack the tricuspid valve. Staphylococcus aureus accounts for between 60% and 90% of cases of endocarditis in intravenous drug users.

12 The endocarditis associated with congenital heart disease (choice A) typically involves either damaged valves or atrial or ventricular septal defects. The tricuspid valve is not particularly vulnerable. Rheumatic fever (choice C) most commonly damages the mitral and aortic valves, and tricuspid damage is usually less severe and seen only when the mitral and aortic valves are heavily involved. Consequently, secondary bacterial endocarditis involving only the tricuspid valve in a patient with a history of rheumatic fever would be unusual. Rheumatoid arthritis (choice D) is not associated with bacterial endocarditis. Systemic lupus erythematosus (choice E) can produce small, aseptic vegetations on valves, but is not associated with bacterial endocarditis.

A 4-year-old boy is brought to the emergency room in extreme respiratory distress, with a temperature of 103.8 degrees Fahrenheit. He is drooling and has difficulty swallowing, and on physical examination, inspiratory stridor is noted. A lateral x-ray shows swelling of the epiglottis. He has had no previous vaccinations. Which of the following agents is the most likely cause of these symptoms? A. Haemophilus influenzae B. Klebsiella pneumoniae C. Legionella pneumophila D. Mycoplasma pneumoniae E. Streptococcus pyogenes Explanation: The correct answer is A. Epiglottitis is the most common disease of the upper respiratory tract produced by Haemophilus influenzae type b, a gram-negative encapsulated rod. It is also a common cause of otitis media in children and may cause bronchitis, bronchiolitis, and pneumonia in adults. The incidence of serious disease caused by Haemophilus influenzae type b has decreased greatly with the introduction of an effective vaccine. The vaccine is composed of the H. influenzae type b capsular polysaccharides coupled to a carrier molecule, given to children between 2 and 15 months of age. The patient had not received the Hib conjugate vaccine and therefore was susceptible to this organism. Klebsiella pneumoniae(choice B) causes pneumonia and pulmonary abscesses, but is not considered to be a pathogen in the upper respiratory tract. Legionella pneumophila(choice C) causes pneumonia in man. The disease may be mild (an atypical pneumonia) or a fulminating disease with a high mortality (30%). Mycoplasma pneumoniae(choice D) causes community-acquired atypical pneumonia. It is the most common cause of pneumonia in young adults. Streptococcus pyogenes(choice E) is the most common cause of pharyngitis; however, this patient's presentation strongly suggests epiglottitis.

A 15-year-old boy presents to his physician with several weeks of slowly worsening pruritus of both of his feet. He is otherwise well and taking no medications. On examination, he has bilateral, erythematous, dry scaling lesions that are most obvious in the interdigital web spaces and on the soles. There is no bleeding or exudate. What

13 would most likely be found in a potassium hydroxide A. Branching hyphae with rosettes of conidia B. Branching, septate hyphae C. Budding yeasts D. Hyphae, arthroconidia, and blastoconidia E. Pigmented, septate hyphal fragments F. Short, curved hyphae and round yeasts Explanation: The correct answer is B. This is a case of tinea pedis, or athlete's foot, caused by a variety of dermatophytic fungi, which are easily detected on alkali mounts of scraped skin as colorless, branching hyphae with cross-walls. Hyphae with rosettes of conidia (choice A) describes the environmental/transmission form for Sporothrix schenckii, the agent of rose gardener's disease, which is a subcutaneous mycosis. Budding yeasts (choice C) describes the form found in clinical specimens from patients with sporotrichosis. Hyphae, arthroconidia, and blastoconidia (choice D) would be found in clinical specimens from patients infected with Trichosporon beigelii (white piedra), which is a superficial mycosis of the hair of the head. Pigmented, septate hyphal fragments (choice E) would be found in cases of phaeohyphomycosis, a diverse group of cyst-forming subcutaneous, pigmented (dematiaceous) fungi, rare in the U.S. Short, curved hyphae and round yeasts (choice F), or the "spaghetti and meatball" presentation in clinical specimens, are characteristic of Malassezia furfur, the agent of pityriasis versicolor. (KOH) mount of a scraping of the affected skin?

A 24-year-old AIDS patient develops chronic abdominal pain, low-grade fever, diarrhea, and malabsorption. Oocysts are demonstrated in the stool. Which of the following organisms is most likely to be the cause of the patient's diarrhea? A. Diphyllobothrium latum B. Entamoeba histolytica C. Giardia lamblia D. Isospora belli E. Microsporidia Explanation: The correct answer is D. All of the organisms listed are protozoa. There are two intestinal protozoa specifically associated with AIDS that can cause transient diarrhea in immunocompetent individuals but can cause debilitating, and potentially life-threatening chronic diarrhea in AIDS patients. These organisms are Isospora belli, treated with trimethoprim-sulfamethoxazole (or other folate antagonists) and Cryptosporidium parvum (no treatment presently available).

14 Diphyllobothrium latum(choice A) is the fish tapeworm and occasionally causes diarrhea. Entamoeba histolytica(choice B) and Giardia lamblia(choice C) are both causes of diarrhea, but they are not specifically associated with AIDS. Microsporidia (choice E) are a protozoan cause of diarrhea, but produce spores rather than oocysts.

An African child develops massive unilateral enlargement of his lower face in the vicinity of the mandible. Biopsy demonstrates sheets of medium-sized blast cells with admixed larger macrophages. This type of tumor has been associated with which of the following? A. Epstein-Barr virus and t(8;14) B. Hepatitis B and t(9;22) C. Herpesvirus and CD5 D. Human immunodeficiency virus and CD4 E. Human papillomavirus and t(2;5) Explanation: The correct answer is A. The patient has Burkitt's lymphoma. This type of lymphoma is a high-grade B-cell lymphoma that occurs in endemic form in Africa (it is the most common neoplasm in children in an equatorial belt that includes Africa and New Guinea) and sporadically in the United States and Europe. The sporadic form is often in an abdominal site and occurs in young adults. The African form of Burkitt's lymphoma has been strongly associated with antibodies directed against Epstein-Barr virus; the association is weaker in sporadic cases. A characteristic translocation, t(8;14) (q24.l3;q32.33) has been described. Hepatitis B (choice B) is associated with hepatocellular carcinoma. t(9;22) is the Philadelphia chromosome, which is seen in some cases of CML and AML. Herpesvirus (choice C) does not have a strong tumor association, although a link to cervical cancer has intermittently been proposed. CD5 is a marker seen in small lymphocytic and mantle cell lymphomas. HIV (choice D) is linked to Kaposi's sarcoma (and AIDS). Some patients also develop primary lymphomas (not usually Burkitt's). CD4 is a marker for helper T cells and some T cell lymphomas. Human papillomavirus (choice E) is linked with common warts, genital condylomata, and genital cancers. t(2;5) is linked to anaplastic large cell lymphoma.

A 49-year-old Vietnamese man is diagnosed with tuberculosis. On physical examination, large flocculent masses are noted over the lateral lumbar back, and a similar mass is located in the ipsilateral groin. This pattern of involvement strongly suggests an abscess tracking along the A. adductor longus B. gluteus maximus C. gluteus minimus D. piriformis

15 E. psoas major Explanation: The correct answer is E. This is the classic presentation of a psoas abscess. This clinical entity was formerly a fairly common complication of vertebral tuberculosis, but is now rare in clinical practice in this country. The psoas muscle is covered by a fibrous sheath known as the psoas fascia. This sheath is open superiorly, permitting an infection involving the soft tissues around the spine to enter the sheath, then track down to the groin. The adductor longus (choice A) is a muscle of the anterior thigh, and is not related to the lumbar portion of the back. The gluteus maximus (choice B) gluteus minimus (choice C) and piriformis (choice D) are muscles of the buttock with no relationship to the groin.

A 36-year-old man with AIDS develops right-sided weakness involving the lower, but not the upper, limb. MRI scans reveal a ring-enhancing lesion within the white matter of the left frontal lobe. A biopsy shows coagulative necrosis of brain parenchyma with macrophage-rich chronic inflammatory infiltration admixed with microscopic cysts that contain characteristic bradyzoites. Which of the following is the most common source of this type of infection? A. Anopheles mosquitoes B. Bird droppings C. Cats D. Cooling systems E. Washbasins Explanation: The correct answer is C. Clues to the correct answer are the underlying disease (AIDS), which predisposes to opportunistic infections, the typical MRI appearance of the lesion (ring-enhancing mass), and the histopathologic features (presence of encysted bradyzoites). In short, this patient has cerebral toxoplasmosis, which represents one of the most common opportunistic infections in AIDS. Toxoplasma gondii is a protozoon that infects humans who ingest the oocysts from cat feces or incompletely cooked lamb or pork. Only immunodepressed patients and fetuses are vulnerable to this infection. In the fetus, toxoplasmosis causes extensive damage to brain parenchyma and retina. Toxoplasmosis associated with AIDS manifests with necrotizing lesions surrounded by chronic inflammation. A ring-enhancing lesion is a mass that contains a rim of contrast enhancement (bright signal on MRI) surrounding a dark core corresponding to central necrosis. In AIDS, the most frequent causes of a ring-enhancing lesion are primary brain lymphoma and toxoplasmosis. Anopheles mosquitoes (choice A) transmit malaria parasites. Cerebral malaria is caused by Plasmodium falciparum, which is able to adhere to endothelial cells of small cerebral vessels. Vascular occlusion ensues, resulting in numerous small infarcts. Bird droppings (choice B) represent the vehicle of infection for two of the most common opportunistic fungal infections affecting immunocompromised patients: cryptococcosis and histoplasmosis. Cryptococcus neoformans causes meningoencephalitis, not intracerebral necrotic lesions. Histoplasma capsulatum rarely affects the brain. These fungi can be visualized in tissue sections by silver stains. Cooling systems (choice D) may harbor Legionella pneumophila, spreading the bacteria in aerosolized form. L.

16 pneumoniae is a gram-negative bacterium that causes Legionnaire's disease, a fatal form of pneumonia that first struck participants at a meeting of the American Legion. It has been reported in immunocompromised patients as well. Washbasins (choice E) frequently contain Pseudomonas aeruginosa, which has also been isolated from respirator devices, cribs, and antiseptic-containing bottles. P. aeruginosa tends to affect patients with cystic fibrosis, severe burns, or immune impairment. It may cause bronchopneumonia, osteomyelitis, endocarditis, external otitis, and keratitis, but not cerebral infection.

A 16-year-old girl presents with a painlessly enlarged lymph node in her right axilla. Peripheral blood counts are within normal limits. The lymph node is biopsied, and numerous granulomas filled with neutrophils and necrotic debris are observed. Which of the following organisms could produce this disease? A. Bartonella henselae B. Borrelia burgdorferi C. Chlamydia psittaci D. Coxiella burnetii E. Rickettsia prowazekii Explanation: The correct answer is A.Bartonella henselae is the infective agent of cat scratch disease, which generally presents as regional lymphadenopathy with or without low fevers and headaches. Bartonella is a gram-variable pleomorphic rickettsial organism that is introduced to the skin in a cat bite or scratch. It produces a self-limited granulomatous response in the draining lymph nodes. Borrelia burgdorferi (choice B) is a spirochetal organism that is transmitted by a tick bite (Ixodes spp.), producing Lyme disease. Lyme disease progresses from a skin rash to fevers, headache and pain over about one month. It may produce lymphadenopathy, but is not associated with granuloma formation. Chlamydia psittaci (choice C) infection occurs after contact with infected bird droppings and produces an atypical pneumonia. The central nervous system may also be involved, but lymph nodes are spared. Chlamydia trachomatis is the chlamydial species that typically produces suppurative nodal granulomas (lymphogranuloma venereum). Coxiella burnetii(choice D) infection is transmitted by inhaling dusts or drinking milk from infected mammals, especially sheep and cows. The disease in humans, Q fever, is marked by mild nonspecific symptoms or pneumonia, and may progress to myocarditis or hepatitis. Rickettsia prowazekii(choice E) produces epidemic (louse-borne) typhus, which is transmitted by body lice and produces a rash akin to Rocky Mountain spotted fever. Although the organism may reside in the lymph nodes in dormancy, it does not elicit granuloma formation. A 28-year-old male intravenous drug user presents with a febrile illness that has lasted 2 weeks. He also complains of chills, weakness, dyspnea, cough, arthralgia, diarrhea, and abdominal pain. On examination, a heart murmur is present, and small, tender nodules are found on the finger and toe pads, along with small hemorrhages on the palms and soles. Three sets of blood cultures are obtained from different veins. While awaiting laboratory confirmation, an empiric antibiotic regimen should primarily be directed at which of the following organisms? A. Enterococci

17 B. Pseudomonas aeruginosa C. Staphylococcus aureus D. Streptococcus pneumoniae E. Streptococcus pyrogenes Explanation: The correct answer is C. This patient is presenting with signs and symptoms of acute infective endocarditis (IE). The fact that the patient is an intravenous drug user places him at a very high risk for the development of IE. The factors that determine the clinical presentation of IE include the nature of the organism, the valve infected, and the route of the infection. Although the clinical characteristics of IE can vary, most patients present with a febrile illness lasting several days to 2 weeks. This illness is often accompanied by a variety of nonspecific signs and symptoms such as chills, weakness, dyspnea, cough, arthralgia, diarrhea, and abdominal pain. Heart murmurs occur in approximately 90% of all patients, but may be absent in patients with right-sided infections. Other clinical signs include Osler nodules (purplish or erythematous subcutaneous papules or nodules on the pads of the fingers and toes), Janeway lesions (hemorrhagic painless plaques on the palms and soles), petechiae (small erythematous painless hemorrhagic lesions that may appear anywhere), and splinter hemorrhages (thin linear hemorrhages found under the nail beds of fingers and toes). The diagnosis of IE is dependent on positive blood cultures and echocardiographic evidence of "valvular vegetation" and/or valvular injury, with echocardiography the preferred method of diagnosis. A past medical history of intravenous drug abuse contributes to the diagnosis. The initial treatment measures of IE should be aimed at the stabilization of cardiac and/or respiratory symptoms, if applicable. The second priority is obtaining three blood cultures from different veins. Once the blood cultures are obtained, empiric antibiotic therapy for Staphylococci, Streptococci, and Enterococci should be instituted. Since Staphylococcus aureus accounts for most cases of acute IE, empiric coverage should primarily be directed toward Staphylococcus aureus. Enterococci(choice A) cause a minority of cases of native valve endocarditis, but are not usually involved in endocarditis associated with intravenous drug abuse. Gram-negative organisms such as Pseudomonas aeruginosa(choice B) are rarely the cause of IE. Streptococcus pneumoniae(choice D) and Streptococcus pyogenes(choice E) are not common causes of IE.

A 20-year-old female presents with a two day history of dysuria and increased urinary frequency. She states that she was recently married and was not sexually active prior to the marriage. Physical exam reveals a temperature of 100.7 F with normal vital signs. Gynecological exam reveals no evidence of discharge, vaginitis, or cervicitis. Urinalysis reveals 14 white blood cells per high-powered field with many gram-negative rods. The most appropriate therapy would be A. ampicillin B. ceftriaxone C. fluconazole D. gentamicin E. metronidazole Explanation: The correct answer is A. The patient's presentation is consistent with a simple urinary tract infection; there is a

18 short history of dysuria, increased urinary frequency and the appearance of white blood cells and gram-negative rods in the urine. Urinary tract infections are common in women after they become sexually active. The infection is likely caused by urethral trauma during intercourse, which leads to bacterial contamination of the bladder. Since the majority of these infections are caused by Escherichia coli (a gram-negative rod), the most appropriate therapy would be ampicillin for around 10 days. Ceftriaxone (choice B) is the treatment of choice for uncomplicated infections with N. gonorrhoeae, now that most strains are resistant to penicillin. Intravenous ceftriaxone is a regimen reserved for the treatment of life-threatening infections. Fluconazole (choice C) is indicated for the treatment of vaginal candidiasis. Since there is no vaginal discharge and the patient has gram-negative rods in the urine, a diagnosis of vaginal candidiasis can be excluded. Gentamicin (choice D) would be an inappropriate choice since the majority of urinary tract infections caused by gram negative rods are sensitive to ampicillin and the potential for toxicity secondary to gentamicin is great. Metronidazole (choice E) is an antibiotic typically used in the treatment of Trichomoniasis, Giardiasis, and Gardnerella, as well as serious infections believed to be caused by anaerobic bacteria. Since there is no vaginal discharge and the patient has gram negative rods in the urine, one can conclude that these infections are not present and the patient instead has a urinary tract infection.

A 36-year-old mother of two children presents with a 4-day history of swollen, painful hands. Her wrists and metacarpophalangeal joints are boggy and inflamed bilaterally. Her 5-year-old son had been sent home from school approximately 3 weeks previously with red cheeks and a blotchy rash on his torso. What is the most likely diagnosis? A. Listeriosis B. Lyme disease C. Mumps D. Parvovirus E. Reiter's syndrome Explanation: The correct answer is D. Parvovirus B19 causes erythema infectiosum, or Fifth disease. The 5-year-old boy has the classic "slapped cheek" appearance. Adults typically do not get the facial rash, but have arthralgias and arthritis. The symmetrical distribution of involved joints is similar to that in rheumatoid arthritis. The onset in adults is typically 3 to 4 weeks after exposure. Parvovirus infections may persist in immunosuppressed patients, resulting in red blood cell aplasia. Listeriosis (choice A) is caused by the gram-positive rod Listeria monocytogenes. Meningitis and bacteremia are common clinical manifestations. Elderly, neonates, pregnant women, and those taking steroids have the highest risk for infection. Lyme disease (choice B) is caused by Borrelia burgdorferi. It is the most common vector-borne disease (Ixodes ticks) in the U.S. The incidence is highest in the summer and fall. The white-footed mouse and the white-tailed deer are zoonotic reservoirs. From 3 to 32 days following a tick bite, the patient develops fever, lymphadenopathy, meningismus, and the characteristic rash (erythema migrans). The rash enlarges and resolves over 3 to 4 weeks. Sequelae include arthritis, carditis, and neurologic abnormalities. Mumps (choice C) is caused by a Paramyxovirus. The virus most commonly affects glandular tissue. Parotitis, pancreatitis, and orchitis are characteristic. Mumps meningoencephalitis is one of the most common viral

19 meningitides. Mumps polyarthritis is most common in men between the ages of 20 and 30 years. Joint symptoms begin 1 to 2 weeks after the parotitis subsides and large joints are involved. Reiter's syndrome (choice E) is a seronegative, asymmetric arthropathy predominantly affecting the lower extremities. It may be triggered by a C. trachomatis infection. In addition to the arthritis, patients may have urethritis (which is usually due to chlamydia), conjunctivitis, mucocutaneous disease such as balanitis, oral ulcerations, or keratoderma. Approximately 80% of patients are HLA-B27 positive.

A 28-year-old pregnant woman develops a flu-like illness with fever, headache, myalgia, and back pain. As a complication of the illness, she has a spontaneous abortion. Examination of the abortus demonstrates severe amnionitis. Which of the following organisms would most likely be isolated from the placental membranes? A. Borrelia burgdorferi B. Leptospira interrogans C. Listeria monocytogenes D. Spirillium minus E. Streptobacillus moniliformis Explanation: The correct answer is C. Spontaneous abortion is a problem associated with Listeriosis, caused by Listeria monocytogenes. The pattern of abortions was first recognized in herd animals, notably sheep and cattle, and then listeriosis was later implicated as a cause of spontaneous abortion in pregnant women. Listeriosis can occur in either epidemic (food-borne or hospital-acquired) forms or may be sporadic (noticed in animal or animal product handlers). Soft cheeses like Brie are a particularly common source of food-borne listeriosis. The disease may range in severity from asymptomatic carrier cases, to flu-like illness, to spontaneous abortion or neonatal death, to fatal illness in children or adults secondary to septicemia or meningoencephalitis. Other localized infections can also occur, primarily in the immunosuppressed. The treatment of choice is intravenous administration of ampicillin or penicillin, often in combination with an aminoglycoside. Trimethoprim-sulfamethoxazole has been used successfully in patients with penicillin allergy. Borrelia burgdorferi(choice A) causes Lyme disease. Leptospira interrogans(choice B) causes leptospirosis. Spirillium minus(choice D) is a cause of rat-bite fever. Streptobacillus moniliformis(choice E) is a cause of rat-bite fever.

A 14-year-old boy has just moved with his family from Brazil to the U.S. He starts complaining of shortness of breath and palpitations. Chest x-ray films demonstrate pulmonary congestion, and ECG shows alterations in heart rhythm. Echocardiography reveals biventricular dilatation with massive cardiac enlargement. An endomyocardial biopsy shows diffuse interstitial fibrosis, myocyte necrosis, chronic inflammation, and the presence of intracellular protozoan parasites. The patient may also develop which of the following complications? A. Achalasia B. Chronic arthritis C. Cysts in the brain

20 D. Pleuritis E. Splenomegaly Explanation: The correct answer is A. The patient has myocarditis due to Trypanosoma cruzi. This infectious condition, known as Chagas disease, is endemic in vast areas of South America and is transmitted from person to person by triatomids known as "kissing bugs." Experts assess the number of persons with Chagas disease at about 7 million with about 35 million at risk in South America. T. cruzi is an intracellular protozoon that localizes mainly in the heart and nerve cells of the myenteric plexus, leading to myocarditis and dysmotility of hollow organs, such the esophagus, colon, and ureter. Cardiac involvement manifests with ventricular dilatation and congestive heart failure secondary to myocyte necrosis and fibrosis. Intracellular parasites can be visualized in tissue sections. Chagas disease is a cause of acquired achalasia, in which the distal third of the esophagus dilates because of loss of its intrinsic innervation. A similar pathologic mechanism accounts for megacolon and megaureter in Chagas disease. The remaining choices refer to different infectious conditions that may also involve the myocardium: Chronic arthritis (choice B) is a manifestation of the chronic stage of Lyme disease, which is caused by Borrelia burgdorferi and is transmitted to humans by deer ticks. Skin, CNS, and heart are the main targets of this infection. Cysts in the brain (cysticerci) (choice C) may develop as a consequence of infestation by the tapeworm Taenia solium. Humans acquire this parasite by ingesting the eggs from undercooked pork. Cysticercosis may also affect the heart, skeletal muscle, and skin. Group B coxsackievirus infections cause pleuritis (choice D) and myocarditis, manifesting with fever, chest pain, and, if myocarditis is severe, congestive heart failure. As in any form of viral myocarditis, the myocardium is infiltrated by lymphocytes, but there are no morphologic markers specific for Coxsackievirus infection. Splenomegaly (choice E), often of massive proportions, is seen in patients with malaria. Plasmodium organisms can also invade the myocardium, leading to myocarditis.

A debilitated 72-year-old woman develops dry cough, fever, headache, and muscular pains. She treats herself with aspirin and ampicillin without any improvement. Her children take her to a local hospital, where chest x-ray films reveal scattered opacities, suggestive of interstitial infiltration. Laboratory investigations demonstrate the presence of cold agglutinins. She is treated with erythromycin, and her symptoms rapidly improve. Which of the following is the most likely etiologic agent of this patient's condition? A. Influenza virus B. Mycoplasma pneumoniae C. Pneumocystis carinii D. Respiratory syncytial virus E. Streptococcus pneumoniae Explanation: The correct answer is B. The patient's clinical presentation is typical of primary atypical pneumonia. In contrast to bacterial pneumonia, primary atypical pneumonia presents with the following features:

21 - Caused by M. pneumoniae; less frequently by viruses (influenza, respiratory syncytial virus, adenovirus, rhinoviruses, rubeola and varicella virus), Chlamydia, or Coxiella burnetii - Characterized pathologically by interstitial, rather than intra-alveolar, inflammation - Characterized clinically by nonspecific symptomatology and few "localizing" symptoms Why is M. pneumoniae, and not influenza virus (choice A) or respiratory syncytial virus (choice D), the cause of this patient's pneumonia? First, M. pneumoniae infections are often associated with the appearance of cold agglutinins in the serum, detection of which is diagnostically important. Second, the patient responded quickly to treatment with erythromycin, an antibiotic effective against M. pneumoniae, but obviously not effective in treating viral infections. Pneumocystis carinii(choice C) is a fungal organism causing pneumonia in severely immunocompromised hosts, especially AIDS patients. P. carinii pneumonia (PCP) is characterized by accumulation of a frothy exudate containing numerous organisms within alveolar spaces. Also, P. carinii is not sensitive to erythromycin. The drug of choice for treatment of PCP is trimethoprim-sulfamethoxazole. Streptococcus pneumoniae(choice E) is the usual causative agent of lobar pneumonia, characterized by consolidation of a single lobe due to intra-alveolar acute inflammatory exudation. Lobar pneumonia is more prevalent in young, healthy individuals, whereas primary atypical pneumonia favors old, debilitated patients. S. pneumoniae is highly sensitive to penicillin.

A 25-year-old female presents with a confluent maculopapular rash that began on her face, then spread downward over her trunk. She states that 3 days ago she started having a fever and headache, with bilateral pain associated with the front and back of her neck. She also complains of joint pain. Which of the following diseases does she most likely have? A. Infectious mononucleosis B. Lyme disease C. Roseola D. Rubella E. Rubeola Explanation: The correct answer is D. Rubella, also called German measles or 3-day measles, is a disease caused by a Togavirus, which are small, enveloped, single-stranded, (+) linear RNA viruses. Approximately 40% of patients are asymptomatic or have mild symptoms. In symptomatic patients, the clinical presentation typically consists of an erythematous rash beginning on the head, which spreads downward to involve the trunk, lasting for approximately 3 days. In addition to a transient rash, symptoms include fever, posterior cervical lymphadenopathy, and arthralgias. The greatest danger from rubella is to the fetus. If clinical rubella develops or seroconversion is demonstrated, there is a high risk of congenital abnormalities or spontaneous abortion. The risk varies from 40%–60% if infection occurs during the first 2 months of gestation to 10% by the 4th month. Females of childbearing age should be warned not to become pregnant within 2-3 months from the time of immunization. Mild arthralgias and other symptoms may develop in 25% of immunized women. Enteroviral rashes may mimic rubella and rubeola. Infectious mononucleosis (choice A) is caused by the Epstein-Barr virus, a herpesvirus. Classic findings include fever, exudative pharyngitis, generalized lymphadenopathy, severe malaise (most common complaint), and hepatosplenomegaly. A rash is not a characteristic feature unless the patient has been treated with ampicillin. Lyme disease (choice B) is caused by the spirochete Borrelia burgdorferi. The disease is transmitted by the bite

22 of the tick, Ixodes dammini. Reservoirs in nature include the white-tailed deer and the white-footed mouse. The initial lesion is an annular rash with central clearing and a raised red border (erythema chronicum migrans) at the bite site. The rash is warm, but not painful or itchy. Patients also have fever, malaise, myalgias, arthralgias, headache, generalized lymphadenopathy, and, occasionally, neurologic findings. Roseola (choice C) is caused by human herpesvirus 6. Other names include exanthem subitum or sixth disease. Children have a febrile period of 3–5 days with rapid defervescence followed by an erythematous maculopapular rash lasting 1–3 days. Rubeola (choice E), or regular measles, is a disease caused by a paramyxovirus. Patients present with an upper respiratory prodrome and characteristic oral lesions (Koplik's spots) that precede the rash. The non-pruritic maculopapular rash begins on the face and spreads to the trunk and extremities, including palms and soles. The incubation period is 10–14 days. Patients also have a posterior cervical lymphadenopathy. The virus is not associated with risk to a fetus.

A Michigan fisherman presents with complaints of chronic diarrhea and fatigue. His physician orders a complete blood count with differential, which reveals a megaloblastic anemia. Which of the following organisms is the most likely cause of this patient's problems? A. Clonorchis sinensis B. Diphyllobothrium latum C. Echinococcus granulosus D. Taenia saginata E. Taenia solium Explanation: The correct answer is B. Vitamin B12 deficiency with resulting megaloblastic anemia is specifically associated with infection with the fish tapeworm, Diphyllobothrium latum, found in Scandinavia and the Great Lakes. The adult worm attaches to the small intestinal mucosa and releases eggs into the feces. Crustaceans living in contaminated water ingest free-swimming embryos that hatch from the eggs. Certain species of fish ingest the crustaceans, then humans acquire the infection by ingesting the undercooked or raw fish. Vitamin B12 deficiency can develop because the tapeworm absorbs vitamin B12 from the intestinal lumen and also interferes with ileal B12 absorption. Clonorchis sinensis(choice A) infections produce obstruction of the biliary tract or pancreatic duct and are associated with an increased risk of cholangiocarcinoma. Echinococcus granulosus(choice C) larvae infect humans and produce unilocular cystic lesions in the liver. Echinococcus is prevalent in areas in which dogs are used to help raise livestock. Taenia saginata is the beef tapeworm (choice D); it causes diarrhea but not megaloblastic anemia. Taenia solium(choice E) is the pork tapeworm. It occasionally causes diarrhea, but is better known as the cause of cysticercosis. It does not produce megaloblastic anemia.

A child with sickle cell anemia is seen in a hematology clinic. Her mother states that she has been feeling very tired lately, and may have "come down with a virus." On physical examination, the girl is very pale, and a complete blood count shows severe anemia. A bone marrow aspirate contains no erythroid precursor cells. The girl was probably infected with which of the following viruses?

23 A. Coxsackie virus B. Echovirus C. Hepadnavirus D. Herpes virus E. Parvovirus Explanation: The correct answer is E. Parvoviruses are small single-stranded DNA viruses, of which only serotype B19 is pathogenic for humans. This virus causes three distinct syndromes: a childhood febrile rash known as erythema infectiosum ("Fifth disease"); aplastic crisis in individuals with chronic hemolytic diseases (sickle cell anemia, thalassemia, etc); and congenital infections that can present as stillbirth, hydrops fetalis (analogous to severe Rh incompatibility), or severe anemia. Coxsackie viruses (choice A) usually cause cold-like illness, but can cause herpangina, myocarditis, and meningitis. Echoviruses (choice B) can infect a variety of organ systems (GI, CNS, eyes, heart, respiratory, skin), but are not a cause of aplastic crises. Hepadnavirus (choice C) is the causative agent of hepatitis B. Herpes viruses (choice D) cause a variety of acute to chronic infections including herpes simplex types I and II, chicken pox, chronic herpes zoster, CMV infection, and Epstein Barr virus infections.

A 43-year-old executive presents to a physician with chronic, symmetric polyarthritis involving the knees. The man gives a history of having developed an extensive rash after a deer hunting trip in Connecticut several years earlier. He recalls that he felt "sick" at the time, and developed knee pain that prevented him from climbing stairs for several months, but then partially resolved. Which of the following organisms is most likely etiologically related to the patient's arthritis? A. Fungus B. Gram-negative cocci C. Gram-negative rod D. Gram-positive cocci E. Spirochete Explanation: The correct answer is E. The history is characteristic for Lyme disease, caused by the spirochete Borrelia burgdorferi. The clues are deer, Connecticut, rash, knees, and acute arthritis followed by chronic arthritis. Fungal arthritis (choice A) can be caused by coccidioidomycosis, blastomycosis, sporotrichosis, and candidiasis. N. gonorrhoeae are gram-negative cocci (choice B) that can cause septic arthritis.

24 Haemophilus (young children) and Salmonella (sickle cause septic arthritis. cell patients) are gram-negative rods (choice C) that can

Staphylococcus are gram-positive cocci (choice E) that can cause septic arthritis.

A 39-year-old HIV-positive man is brought into the emergency room after experiencing a seizure witnessed by several friends. The observers relate that the patient suddenly lost consciousness and experienced both leg and arm jerking. The man's tongue has been severely bitten, and loss of bowel and bladder function is evident upon admission. On physical examination, the patient is lethargic, unable to answer simple questions, and has an obvious left-sided hemiparesis. An MRI of the head shows multiple ring-enhancing lesions. Infection with which of the following agents is most likely responsible for this presentation? A. Cryptococcus neoformans B. Herpes simplex C. Mycobacteria tuberculosis D. JC virus E. Toxoplasma gondii Explanation: The correct answer is E. While all five pathogens can cause central nervous system (CNS) manifestations, toxoplasmosis (caused by Toxoplasma gondii) presents with seizures in 15-25% of cases. The disease is spread by ingestion of cysts from undercooked meat or from cat feces. Tachyzoites develop from cysts phagocytized by macrophages, then spread to the brain, muscle, and other tissues, where they encyst and multiply. Primary CNS lymphoma in AIDS patients can present clinically in nearly the same manner as toxoplasmosis, but the course is usually much more indolent. Cryptococcal meningitis (choice A) usually presents as a subacute meningitis with headache, nausea, vomiting, and confusion. Cranial nerve abnormalities are common with cryptococcal infections. Herpes simplex encephalitis (choice B) typically has a subacute onset with headache, meningismus, and personality changes. Mycobacterium tuberculosis(choice C) is a cause of a basilar meningitis, which can present insidiously with headache and mental changes over a week or two, or can present acutely as confusion, lethargy, altered sensorium, and a stiff neck. Cranial nerve palsies, focal cerebral ischemia, and hydrocephalus are characteristic. Progressive multifocal leukoencephalopathy (PML) is a demyelinating disease caused by JC virus (choice D), a human polyomavirus. PML occurs in about 4% of patients with AIDS. The disorder has a slow, insidious onset, in which altered mental status is not a prominent feature.

Which of the following organisms would most likely cause infection after a sterilization procedure that killed vegetative cells but did not kill spores? A. Chlamydia B. Clostridium C. Escherichia

25 D. Pseudomonas E. Streptococcus Explanation: The correct answer is B. Although bacterial spores are extensively discussed in microbiology courses, you should be aware that only Bacillus (aerobic) and Clostridium (anaerobic) species produce spores. This means that the list of diseases related to bacterial spore formation is also small: anthrax (Bacillus anthracis), some forms of food poisoning (Bacillus cereus), botulism (Clostridium botulinum), tetanus (Clostridium tetani), gas gangrene (Clostridium perfringens and others), and pseudomembranous colitis (Clostridium difficile). A newborn in the neonatal intensive care unit becomes tachypneic and irritable. Blood cultures grow a gram-positive coccus in chains that is presumed to be a streptococcus. Which of the following characteristics would help to differentiate Streptococcus agalactiae from Streptococcus pneumoniae? A. Alpha-hemolysis B. Carbohydrate capsule C. Cytochrome enzyme system D. Growth in bile E. Oxacillin sensitivity Explanation: The correct answer is A. Streptococci are usually initially speciated by their hemolytic capacity on sheep blood agar. Beta-hemolytic streptococci include groups A, B, and D. S. agalactiae is the classic group B streptococcus. The non beta-hemolytic streptococci consist principally of the pneumococci and the viridans group. Both S. agalactiae and pneumococcus have a carbohydrate capsule (choice B), an important virulence factor and means of subtyping streptococcal species. None of the streptococci utilize cytochrome enzymes (choice C). They derive all of their energy from the fermentation of sugars to lactic acid. Neither pneumococcus nor S. agalactiae can grow in bile (choice D). This ability is specific for the enterococcus group (group D) of streptococci. Both pneumococci and S. agalactiae are usually treated with penicillin-type antibiotics, although group B streptococci require a penicillinase-resistant type such as oxacillin (choice E).

A 35-year-old man presents to the emergency room with intense back pain. He is hydrated and given pain medication. After several hours he passes a kidney stone. Laboratory analysis of the stone reveals that it is composed of struvite (magnesium ammonium phosphate). Infection with which of the following organisms promotes the production of such stones? A. Escherichia coli

26 B. Proteus mirabilis C. Pseudomonas aeruginosa D. Staphylococcus saprophyticus E. Ureaplasma urealyticum Explanation: The correct answer is B. Proteus species produce urease, which raises the urinary pH and promotes the production of struvite stones. E. coli(choice A) are by far the most common cause of urinary tract infections, but are not responsible for the development of struvite stones. Pseudomonas aeruginosa(choice C) can also cause urinary tract infections, most commonly in hospitalized or immunocompromised patients (especially burn patients, patients on immunosuppressives/antimicrobials, and cystic fibrosis patients). It does not predispose to struvite stones. Staphylococcus saprophyticus(choice D) is a common cause of urinary tract infections in sexually active young women. Ureaplasma urealyticum(choice E) can produce urease (like Proteus), but they are responsible for urethritis, not stones.

A 54-year-old truck driver in Oklahoma collides with a deer on the highway. He takes the deer home and butchers it. Five days later, he develops fever, non-productive cough, shortness of breath, and generalized lymphadenopathy. No skin lesions are noted. What is the most likely diagnosis? A. Anthrax B. Leptospirosis C. Pasteurella multocida infection D. Plague E. Tularemia Explanation: The correct answer is E. Tularemia is caused by Francisella tularensis. Fifty percent of the cases have been reported in Missouri, Arkansas, and Oklahoma. Infection can be transmitted by tick bites or by handling animal carcasses such as rabbits, squirrels, beavers, muskrats, and deer. The diagnosis is suggested by the history of exposure, clinical progression, and generalized lymphadenopathy. The ulceroglandular form of tularemia is the most common clinical presentation. Anthrax (choice A) is caused by Bacillus anthracis. Typically, infection with B. anthracis causes a localized skin lesion at the site of inoculation that develops into a black eschar surrounded by draining lymphadenitis. Woolsorter's disease is the inhalation form of anthrax. Leptospirosis (choice B) is an infection caused by contact with urine from infected animals such as rats and dogs. L. interrogans is the most common isolate. The disease is biphasic, with the leptospiremic phase characterized by abrupt-onset headache (98%), fever, chills, conjunctivitis, severe muscle aches, gastrointestinal symptoms, changes in sensorium, rash, and hypotension. This phase lasts from 3 to 7 days.

27 The immune stage occurs after a relatively asymptomatic period of 1 to 3 days and is characterized by recurrence of fever and generalized symptoms. Meningeal symptoms often develop during the immune period. In more serious cases, hepatic dysfunction and renal failure may develop. Pasteurella multocida(choice C) is associated with dog and cat bites. It causes a rapidly progressing cellulitis, bacteremia, and, occasionally, infective endocarditis. Plague (choice D) is caused by Yersinia pestis. It is enzootic in the southwestern United States. Rats and fleas are the vectors. Clinical presentations include lymphadenopathy with septicemia or pneumonia (which has the highest case-fatality rate).

A 67-year-old black man with a history of glucose-6-phosphate dehydrogenase (G-6-PD) deficiency presents with fever, irritative voiding symptoms, and perineal pain. Rectal examination is remarkable for a boggy, exquisitely tender prostate. A urine Gram's stain is positive for gram-negative rods. The risk for development of hemolytic anemia is highest if he receives high-dose, 21-day therapy with A. ampicillin B. cefaclor C. ciprofloxacin D. sulfamethoxazole/trimethoprim E. tetracycline Explanation: The correct answer is D. The patient has acute prostatitis, which is characterized by fever, chills, and dysuria, with a swollen, extremely tender prostate on rectal exam. The urine Gram's stain and culture will generally be positive. The treatment regimen for this bacterial infection is typically a 21-day course of ampicillin, a fluoroquinolone, or sulfamethoxazole/trimethoprim (SMX-TMP). G-6-PD deficiency is an X-linked recessive disorder affecting 10% to 15% of American black males. The medications most commonly associated with the induction of hemolytic anemia in deficient patients are sulfonamides, nitrofurantoin, dapsone, primaquine, and quinine. The sulfamethoxazole in the SMX-TMP combination is a sulfonamide, and can produce hemolytic anemia in patients with G-6-PD deficiency. Ampicillin (choice A) is a broad-spectrum penicillin antibiotic commonly used in the treatment of infections in the genitourinary, respiratory, or GI tracts, as well as in the skin and soft tissues. Cefaclor (choice B) is a second-generation cephalosporin indicated for a variety of bacterial infections, including those of the respiratory and GI tracts. Ciprofloxacin (choice C) is a fluoroquinolone commonly used in the treatment of serious infections caused by gram-negative organisms. Tetracycline (choice E) is most commonly used in the treatment of acne vulgaris and gonococcal infections. A 25-year-old man presents with a high fever and generalized malaise. His condition deteriorates so rapidly that his friends decide to take him to the emergency department 24 hours after the onset of symptoms. He has a history of intravenous drug abuse. A test for anti-HIV antibodies is negative. Physical examination reveals a systolic murmur, and echocardiography shows bulky vegetations attached to the tricuspid valve leaflets. Which of the following microorganisms will be most likely be isolated from this patient's blood cultures? A. Candida albicans

28 B. Hemophilus influenzae C. Staphylococcus aureus D. Staphylococcus epidermidis E. Viridans (α-hemolytic) streptococci Explanation: The correct answer is C. The patient has a fever and is extremely ill. The most important clue to the diagnosis is the presence of bulky vegetations on the tricuspid valve, indicating that he has infective endocarditis. On the basis of the rapid clinical course, this is likely a case of acute infective endocarditis. The diagnosis of this condition must be confirmed by blood cultures, which are also necessary to determine bacterial antibiotic sensitivity. S. aureus (commonly present on the skin) is the most frequent etiologic agent of infective endocarditis in intravenous drug abusers. It commonly affects the tricuspid valve. Because of its high virulence, S. aureus-related endocarditis follows an acute course and may lead to death within a few days. The causative agents of infective endocarditis differ depending on host factors. Fungal organisms, such as Candida albicans(choice A), may cause infective endocarditis in severely immunosuppressed patients, such as those with AIDS. A minority of cases of infective endocarditis are caused by a number of normal commensals in the oral cavity, i.e., the "HACEK" group: Hemophilus(choice B), Actinobacillus, Cardiobacterium, Eikenella, and Kingella. S. epidermidis(choice D) and other coagulase-negative staphylococci tend to produce endocarditis in recipients of prosthetic valves. Viridans streptococci (choice E) are the most frequent agents causing endocarditis in previously abnormal valves, such as those damaged by rheumatic disease, or congenitally abnormal valves. Coagulase-negative staphylococci and viridans (α-hemolytic) streptococci are less virulent than S. aureus and are thus associated with a subacute (more prolonged) clinical course and a better prognosis.

A newborn infant develops respiratory distress shortly after delivery and is taken to the neonatal intensive care unit for observation. The next day she becomes febrile with persistent dyspnea and coughing. Sputum recovered by the respiratory therapist reveals numerous neutrophils and gram-negative rods that grow as pink colonies on MacConkey's agar. The infective organism most likely belongs to which of the following genera? A. Escherichia B. Proteus C. Pseudomonas D. Salmonella E. Shigella Explanation: The correct answer is A. Essentially, this question tests your understanding of MacConkey's agar and your knowledge of the biochemical characteristics of the Enterobacteriaceae. MacConkey's agar is used to identify organisms that ferment lactose, and consequently grow as pink colonies. Escherichia coli is one of several lactose-fermenting Enterobacteria, along with Serratia, Klebsiella, and Citrobacter species. Neonatal pneumonia due to aspirated E. coli during delivery is not uncommon, and it should be treated with a third-generation

29 cephalosporin and/or aminoglycoside. Proteus species (choice B) are motile, non-lactose fermenting bacteria that characteristically swarm on an agar plate. Proteus infections generally affect the genitourinary tract, but Proteus is an important nosocomial pathogen. Pseudomonas species (choice C) are normally widespread in nature, but may form a small portion of the bowel flora. Pseudomonas utilizes cytochrome oxidase for energy production; it does not ferment any sugars. Salmonella species (choice D) are non-lactose fermenters that may produce inflammatory diarrheas. They are normally distinguished from the other Enterobacteriaceae by their ability to produce acid and gas from dextrose-containing media. Shigella species (choice E) are non-motile, non-lactose fermenting bacteria that live exclusively in the gastrointestinal tract of primates. They typically produce dysentery.

Five days after returning to his military base in South Carolina after survival training in the nearby countryside, an 18-year-old recruit reports to the infirmary complaining of a headache. Physical examination reveals a fever, but no other abnormalities are noted. A few days later he returns to the infirmary with a maculopapular rash involving the hands and feet. The rash then spreads centripetally to involve the trunk. Which of the following diseases should be suspected? A. Chickenpox B. German measles C. Measles D. Mumps E. Rocky Mountain spotted fever Explanation: The correct answer is E. Take rashes involving the palms and soles (otherwise unusual sites) very seriously: only a small number of infections can cause this pattern, including Rocky Mountain spotted fever, meningococcemia, and secondary syphilis. Rocky Mountain spotted fever is caused by the rickettsia R. rickettsii, and is found throughout the United States, particularly in south central and eastern portions (not the Rocky Mountains). 3-12 days after a tick bite, patients develop malaise, frontal headache, and fever. Several days later, the rash described in the question stem develops. Other manifestations can include hepatosplenomegaly, thrombocytopenia, and (potentially fatal) disseminated intravascular coagulation. Chickenpox (choice A), or varicella, is characterized by maculopapules that evolve into vesicles over hours to days, then eventually form crusts. Typical lesions first appear on the trunk and face and rapidly spread to involve other areas. The maculopapular rash of German measles (choice B), or rubella, usually begins on the face, then spreads down the body. Although the maculopapular rash of measles (choice C), or rubeola, can include the palms and soles, it typically begins along the hairline in frontal and temporal regions, then spreads down the trunk to the limbs. Mumps (choice D) is characterized by fever, malaise, parotitis, and orchitis, but not a rash.

30 A 47-year-old grocer complains of diarrhea and painful arthritis. Physical examination is remarkable for lymphadenopathy and weight loss. Biopsy of his small bowel reveals PAS-positive macrophages within the lamina propria. Electron microscopic examination of the macrophages reveals small rod-shaped structures. These structures are most likely A. Clostridium B. Enterotoxigenic E. coli C. Isospora D. Salmonella E. Tropheryma Explanation: The correct answer is E. For many years, Whipple's disease was suspected of having a bacterial etiology because bacterial forms could be seen on electron microscopy. However, the identity of the causative agent remained elusive. It has recently been found to be a bacterium which is now named Tropheryma whippelii. Clostridium difficile(choice A) causes pseudomembranous colitis, generally after antibiotic administration. Enterotoxigenic E. coli(choice B) is associated with tropical sprue. Isospora bella(choice C) is a cause of diarrhea in AIDS patients. Salmonella sp.(choice D) can cause diarrheal illness after ingestion of contaminated poultry or beef.

A 33-year-old G1P0 female at 6 months gestation returns from a visit to her parents house in Arizona. Approximately 7 days following her return, she develops axillary lymphadenopathy and a low-grade fever. Her physician notices a small papule and healing scratch on her arm on the affected side. The patient states that she has pet birds at home, and there was a new kitten at her mother's house. She does not remember receiving the scratch. Which of the following organisms is most likely responsible for this illness? A. Bartonella henselae B. Coccidioides immitis C. Cryptococcus neoformans D. Histoplasma capsulatum E. Pasteurella multocida Explanation: The correct answer is A.Bartonella henselae is a gram-negative bacillus that is the causative agent of cat scratch disease. Cats, especially kittens, harbor the organism. It is usually inoculated through an easily forgotten scratch. A pustule forms at the site within days to weeks, accompanied by tender regional adenopathy. Microscopically, granulomas with central necrosis and organisms are observed. Normally, treatment is not required because this disease is benign and self-limited, but immunosuppressed and pregnant patients are more susceptible to dissemination. Treatment is with ciprofloxacin or doxycycline. Aminoglycosides or erythromycin have also been employed.

31 Coccidioidomycosis is a disease caused by the fungus C. immitis(choice B). Spores of the organism are inhaled, usually from the soil. These spores enlarge to sporangia, which burst and spread. The disease is characterized by pulmonary symptoms, but is usually self-limited. This organism is common in arid desert states like Arizona, and can affect people after brief visits. Cryptococcus neoformans(choice C) is a fungus found worldwide. The main reservoir is pigeon droppings. The small cryptococci are inhaled into the lungs, but do not usually cause disease in immunocompetent individuals. In the immunosuppressed patient, Cryptococcus can cause pulmonary and CNS disease, especially meningitis. Histoplasma capsulatum(choice D) is an infection acquired by breathing dust that contains bird/bat droppings that contain spores of the organism. The primary infection is in the lungs, and may be latent, or brief and self-limited. Symptoms include fever, cough and sweats. In the United States, it is common in the Ohio-Mississippi valley. Pasteurella multocida(choice E) is a gram-negative rod spread via dog and cat bites. The infection is characterized by rapidly evolving cellulitis starting at the site of the bite. There was no indication in this patient of an animal bite.

A 33-year-old woman begins seeing a new boyfriend. Several weeks later, a week after her menstrual period, the woman develops a painful, swollen right knee. The most likely causative organism has which of the following characteristics? A. Both hyphae and spores B. Can live within neutrophils C. Cannot make ATP D. No true cell wall E. Stains with silver stains Explanation: The correct answer is B. The causative organism is Neisseria gonorrhoeae, which is a sexually transmitted gram-negative coccus that can live in phagocytic vacuoles in neutrophils and macrophages. The organism is the most common cause of septic arthritis in otherwise healthy, sexually active adults. Hyphae and spores (choice A) would be features of Candida, which usually causes arthritis as a rare complication of systemic candidiasis in immunocompromised patients. The Chlamydia are intracellular organisms that cannot make ATP (choice C) and do not usually cause septic arthritis. Mycoplasma do not have true cell walls (choice D) and do not usually cause septic arthritis. Syphilis is caused by a spirochete that stains with silver stains (choice E). Secondary syphilis can (uncommonly) cause an acute arthritis, but the interval between acquiring the infection and developing the arthritis is usually months to years.

A 32-year-old man presents to the emergency room with a severe headache. Nuchal rigidity is found on physical examination. Lumbar puncture demonstrates cerebrospinal fluid with markedly increased lymphocytes. Other cell populations are not increased. Which of the following agents is the most likely cause of his symptoms?

32 A. Escherichia coli B. Haemophilus influenzae C. Herpes virus D. Mycobacterium tuberculosis E. Treponema pallidum Explanation: The correct answer is C. The clinically suspected diagnosis is meningitis, which is confirmed by the abnormal cerebrospinal fluid. The markedly increased lymphocytes suggests acute lymphocytic meningitis, which is distinguished from acute pyogenic meningitis (increased neutrophils as well as lymphocytes). Acute lymphocytic meningitis is usually viral in origin. Among the many viruses that have been implicated, mumps, herpes, Epstein-Barr, echovirus, and Coxsackie virus are the most common. Escherichia coli(choice A) and Haemophilus influenzae(choice B) cause acute pyogenic meningitis. Mycobacterium tuberculosis(choice D) and Treponema pallidum(choice E) cause chronic meningitis. A febrile 12-year-old child presents with severe right lower quadrant pain that is interpreted by the attending physician as acute appendicitis. The patient has also been complaining of joint pain. At laparotomy, the surgeon notes that the appendix is normal; however, the mesenteric lymph nodes are markedly enlarged and contain focal areas of microabscess formation on cut section. This patient is most likely A. an asthmatic B. deficient in C1 esterase inhibitor activity C. HLA-B27 positive D. leukopenic E. serologically positive for toxoplasmosis Explanation: The correct answer is C.Yersinia enterocolitica is the pathogen producing this clinical syndrome. Yersinia is transmitted to patients via the oral route, via contaminated blood products, or by cutaneous inoculation. Patients developing iron overload because of multiple transfusions (i.e., thalassemia patients) are at increased risk of Yersinia infections because some strains are unable to synthesize bacterial iron chelators called siderophores. They can, however, use host-chelated iron stores or the drug deferoxamine (a siderophore produced by Streptomyces pilosus). Yersinia is associated with reactive arthritis following an infection by an enteropathogenic organism. Most patients who develop arthritis express HLA-B27. A history of asthma (choice A) is not associated with Yersinia infections. Deficiency in C1 esterase inhibitor activity (choice B) produces the syndrome of angioedema. This is an autosomal dominant trait associated with a deficiency of the serum inhibitor of the activated first complement component. The patients have multiple episodes of edema, affecting skin and mucosal surfaces such as the larynx and the GI tract. Leukopenia (choice D) is not associated with the clinical scenario in the question. Normal-to-elevated leukocyte counts are the rule in Yersinia infection.

33 Serological positivity for toxoplasmosis (choice E) is not suggested because toxoplasmosis produces a different clinical picture. Toxoplasmosis can be acquired transplacentally with infected infants exhibiting a wide array of symptoms depending on when the mother was infected during gestation. Toxoplasmosis is also seen in immunocompromised patients such as those with AIDS. In this group of patients, the main organ system involved is the central nervous system.

A 16-year-old girl presents to her physician complaining of redness and a yellowish discharge from her left eye for the past 2 days. She reports minimal crusting upon awakening and denies eye pain or previous trauma. Upon examination, there is diffuse conjunctival hyperemia associated with a mucoid discharge. The pupils dilate normally, although there is mild photophobia in the affected eye. Gram's stain of the exudate shows pleomorphic, gram-negative bacilli. To culture this organism with whole blood agar, which of the following must be used? A. Antibiotics to inhibit normal flora B. Cold temperature C. Egg yolk cholesterol D. Reduced oxygen tension E. Staphylococcus aureus Explanation: The correct answer is E. The patient is suffering from conjunctivitis caused by Hemophilus, a fastidious organism requiring factors V (nicotinamide adenine dinucleotide) and X (hematin), which must be released from whole blood preparations to be accessible to the organism. This can be done by gentle heating, which lyses the red blood cells, or by co-culture with Staphylococcus aureus, which is beta hemolytic and lyses the red blood cells to provide the factors Hemophilus needs for growth. This is called the satellite phenomenon. Antibiotics (choice A) would not be necessary since, with proper culture techniques, no normal flora should be available to confuse the diagnosis from this site. Cold temperature (choice B) would not be necessary because Hemophilus grows at normal body temperature. Listeria is the genus commonly mentioned for which a cold temperature is necessary. Cholesterol (choice C) is necessary for the growth of Mycobacteria, but not Hemophilus. Low oxygen (choice D) tension is necessary for the growth of the microaerophiles (such as Campylobacter, Helicobacter, and Borrelia), but is not a condition for growth of Hemophilus. A baby born at 32 weeks gestation with Apgar scores of 2 and 7 was placed in the neonatal intensive care unit. She developed a fever and leukocytosis; lumbar puncture revealed pleocytosis with increased protein, decreased glucose, and gram-positive rods. Which one of the following organisms was most likely isolated from the CSF? A. Escherichia coli B. Listeria monocytogenes C. Neisseria meningitidis D. Streptococcus agalactiae E. Streptococcus pneumoniae

34 Explanation: The correct answer is B. The three major causes of neonatal meningitis are group B streptococci (Streptococcus agalactiae; choice D), Escherichia coli(choice A), and Listeria monocytogenes. All can be found in the vaginal tract of normal women and may contaminate the infant during passage through the birth canal. They colonize the upper respiratory tract and can cause pneumonia, septicemia, and/or meningitis in the neonate. They are readily distinguished on morphologic grounds; the streptococci are gram-positive cocci in chains, E. coli is a gram-negative rod, and L. monocytogenes is a gram-positive pleomorphic rod. There are other gram-positive rods that resemble Listeria (e.g., the diphtheroid bacilli found in the upper respiratory tract and on the skin), hence a motility test is done to confirm the identification. L. monocytogenes is motile at room temperature but not when grown at 37 degrees C. L. monocytogenes is also associated with drinking unpasteurized milk. Neisseria meningitidis(choice C) is the most common cause of meningitis in school-age children and young adults. It is a fastidious, non-motile, gram-negative diplococcus that would be a very rare cause of meningeal disease in very young patients, such as this one. Streptococcus pneumoniae(choice E) is a gram-positive coccus that grows in pairs and short chains. It is the number one cause of pneumonia, septicemia, and meningitis in the elderly. There is a vaccine composed of the capsular carbohydrate of 23 serotypes of this organism that is routinely given to individuals over the age of 60, as well as to individuals with splenic abnormalities (e.g., sickle cell disease) who are at increased risk for the development of pneumococcal sepsis. A 27-year-old intravenous drug user presents with difficulty swallowing. Examination of the oropharynx reveals white plaques along the tongue and the oral mucosa. Which of the following best describes the microscopic appearance of the microorganism responsible for this patient's illness? A. Budding yeast and pseudohyphae B. Encapsulated yeast C. Mold with nonseptate hyphae D. Mold with septate hyphae Explanation: The correct answer is A. This patient has Candida esophagitis. Any time a patient presents with dysphagia or odontophagia and you note white plaques in the oropharnyx (thrush), you can assume that the Candida is affecting the esophagus as well. The fact that the patient is an IV drug user makes an opportunistic infection such as Candida more likely. Now comes the hard part–remembering the morphology of Candida. Candida appears as budding yeast with pseudohyphae in vivo. The other answer choices represent the morphology of other important opportunistic fungi: Cryptococcus are encapsulated yeast (choice B). You should think about Cryptococcus neoformans when you're presented with an immunocompromised patient with neurological symptoms. The classic clue is the presence of encapsulated organisms observable in an India ink preparation. Mucor and Rhizopus are molds with nonseptate hyphae (choice C). You should think about Mucor when you are presented with a diabetic (especially ketoacidosis) or a leukemic patient with a severe sinus infection. Aspergillus is a mold with septate hyphae (choice D). In immunocompromised patients, aspergillosis can present with acute pneumonia, often with cavitation (aspergillomas = fungus balls in the lungs).

35 A 24-year-old woman in her third trimester of pregnancy presents with urinary frequency and burning for the past few days. She denies fever, nausea, vomiting, or chills. She takes no medications besides prenatal vitamins and is generally in good health. Physical examination is remarkable for mild suprapubic tenderness, and a urine dipstick is positive for white blood cells, protein, and a small amount of blood. Culture produces greater than 100,000 colonies of gram-negative bacilli. Which of the following attributes of this uropathogenic organism is most strongly associated with its virulence? A. Bundle-forming pili B. GVVPQ fimbriae C. Heat labile toxins D. Heat stable toxins E. P pili F. Type 1 pili Explanation: The correct answer is E. Urinary tract infections are the most common bacterial infections encountered during pregnancy, and Escherichia coli is the most commonly isolated organism. 70% of cases in the U.S. are caused by P pili-positive strains. Bundle-forming pili (choice A) are found in enteroaggregative E. coli (EAEC). GVVPQ fimbriae (choice B) are found in EAEC. Heat labile toxins (choice C) are pathogenic factors in enterotoxic strains (ETEC). Heat stable toxins (choice D) are pathogenic factors in ETEC or EAEC. Type 1 pili (choice F) are a major pathogenic factor in ETEC. A febrile 23-year-old college coed presents with fatigue and difficulty swallowing. Physical exam reveals exudative tonsillitis, palatal petechiae, cervical lymphadenopathy, and tender hepatosplenomegaly. A complete blood count reveals mild anemia, lymphocytosis with about 30% of the lymphocytes exhibiting atypical features, and a mild thrombocytopenia. Coombs' test is positive. Which of the following is the most likely complication of this syndrome? A. Acute cholecystitis B. Ascending cholangitis C. Diarrhea D. Immune complex vasculitis E. Splenic rupture Explanation: The correct answer is E. The syndrome represented by the clinical vignette is infectious mononucleosis. Epstein-Barr virus (EBV) is the usual cause of heterophile-positive infectious mononucleosis; cytomegalovirus is

36 responsible for a minority of cases. Rarely, splenic rupture requiring splenectomy can result from splenomegaly and capsular swelling, usually occurring during the 2nd and 3rd weeks of the illness. Acute cholecystitis (choice A) is not associated with infectious mononucleosis. The most frequently isolated pathogens are E. coli, Klebsiella spp., group D Streptococcus, Staphylococcus spp., and Clostridium spp. Ascending cholangitis (choice B) is not associated with infectious mononucleosis. Cholangitis usually presents with biliary colic, jaundice, and spiking fever with chills (Charcot's triad). Blood cultures are usually positive (E. coli is a common isolate), with an accompanying leukocytosis. AIDS-related cholangitis has been reported, presenting with abdominal pain and obstructive liver symptoms. Potential etiologic agents include Cytomegalovirus, Cryptosporidium parvum, and Microsporidia, including Enterocytozoon cuniculi. Diarrhea (choice C) is not usually produced by infectious mononucleosis. Immune complex vasculitis (choice D) is not associated with infectious mononucleosis.

A 37-year-old newly married man presents with multiple blister-like lesions on the glans of his penis, appearing over the past 2 days. On questioning, he recalls similar episodes over the past 2 years. Examination is remarkable for tender, 3-4 mm vesicular lesions on the shaft of his penis with no apparent crusting, drainage, or bleeding. There is also slight bilateral inguinal adenopathy. During the asymptomatic period between outbreaks, where would the causative agent likely have been found? A. Fibroblasts B. Lymphocytes C. Macrophages D. Mucoepithelial cells E. Neurons of the sacral ganglia F. Neurons of the trigeminal ganglia Explanation: The correct answer is E. This is a classic example of an infection with herpes simplex virus (probably type 2). This agent causes lytic infections in mucoepithelial cells. It is then retrogradely transported into neurons of the sacral ganglia, where it lays dormant during the asymptomatic phase of the disease. Fibroblasts (choice A) may be infected by cytomegalovirus, another herpesvirus, but this produces a distinctive mononucleosis-like syndrome in most normal individuals. Lymphocytes (choice B) and macrophages (choice C) may be infected by herpes simplex type 2 but are not the site of residence of the virus during quiescent periods. Infection of mucoepithelial cells (choice D) by herpes simplex produces vesicular-like lesions on the mucous membranes during symptomatic periods. Neurons of the trigeminal ganglia (choice F) may be latently infected with herpes simplex type 1. This agent is a possible cause of genital ulcers, but is usually associated with perioral lesions (cold sores).

A 67-year-old woman in a skilled nursing facility complains of flu-like symptoms. After several days, she develops high fever, dyspnea, and a productive cough. The nurses also notice mental status changes, and she is transported to the nearby community hospital. A chest x-ray shows a cavitary lesion in her left lung. Which of the

37 following organisms would most likely be identified A. Candida albicans B. Klebsiella pneumoniae C. Pneumocystis carinii D. Staphylococcus aureus E. Streptococcus pneumoniae Explanation: The correct answer is D. The woman has developed a pulmonary abscess, as evidenced by the chest radiograph. Of the organisms listed, Staph. aureus is the most likely cause of bacterial pneumonia complicated by abscess formation. Bacteria commonly producing pneumonia developing in the context of influenza include Staphylococcus aureus, Haemophilus influenzae, and Streptococcus pneumoniae, but Streptococcus pneumoniae is not a frequent cause of lung abscesses. Candida albicans(choice A) would not be a likely cause of this woman's abscess, or of the pneumonia that preceded it. Klebsiella pneumoniae(choice B) is more likely in men who are alcoholic or diabetic, or suffering from chronic obstructive lung disease. Pneumocystis carinii(choice C) is associated with pneumonia in immunocompromised hosts, but there is no indication of immune compromise in this patient. Streptococcus pneumoniae(choice E) is only rarely associated with lung abscess. from examination of her sputum?

A man presents to a dermatologist because of a severe mucocutaneous rash that involves most of his body, including his palms and soles. Questioning reveals that he is a merchant marine who several months previously had an encounter with a prostitute in Southeast Asia. Which of the following is the most likely causative agent of this rash? A. Herpes simplex I B. Herpes simplex II C. HIV D. Neisseria gonorrhoeae E. Treponema pallidum Explanation: The correct answer is E. The rash described is that of secondary syphilis, caused by Treponema pallidum. Involvement of palms and soles by a rash is unusual, and secondary syphilis should come to mind. Not all patients with secondary syphilis have a severe form of the rash, and consequentially some cases are missed. Primary syphilis takes the form of a painless, button-like mass called chancres. Tertiary syphilis, which is now rare, has a propensity for involving the aorta and central nervous system and can also cause "gummas" (granulomatous-like lesions) in many sites, notably including liver and bone.

38 Herpes simplex I (choice A) usually causes perioral vesicular lesions. Herpes simplex II (choice B) usually causes genital vesicular lesions. HIV (choice C) does not itself cause a rash, although co-infection with other organisms can result in a rash. Neisseria gonorrhoeae(choice D) does not typically cause a rash.

A child is brought to the pediatrician because of perianal itching, which has been disturbing the child's sleep. Physical examination reveals scaly skin in the perianal region. The physician instructs the parents to place transparent tape on the perianal region the next morning, and then return to the office. In the office, the tape is transferred to a microscope slide, and low-power examination of the tape reveals oval eggs that are flattened along one side. The microorganism most likely responsible for the child's symptoms is A. Ancylostoma duodenale B. Ascaris lumbricoides C. Enterobius vermicularis D. Necator americanus E. Trichuris trichiura Explanation: The correct answer is C.Enterobius vermicularis (pinworm) inhabits the large intestine, and the gravid females migrate at night into the perianal region to lay eggs, which can be identified by placing cellophane tape on the perianal skin and then looking at the tape with a microscope. Enterobiasis is characterized by extreme pruritus and is very contagious, transmitted by ingestion of the eggs of the organism. The infection is easily treated with a single dose of mebendazole. Ancylostoma duodenale(choice A) and Necator americanus(choice D) are hookworms, and typically cause anemia. Ascaris lumbricoides(choice B) is a large, intestinal roundworm that is very common, especially in the tropics. Infection may be asymptomatic or can cause abdominal symptoms. Trichuris trichiura(choice E) is the whipworm, which can cause rectal prolapse with heavy infestations.

One week following a visit to the woods along an Eastern seaboard beach, a 50-year-old woman develops fever, headache, chills, and fatigue. A blood smear demonstrates protozoa within erythrocytes. Which of the following is the most likely pathogen? A. Babesia microti B. Leishmania donovania C. Plasmodium falciparum D. Plasmodium vivax E. Trypanosoma cruzi

39 Explanation: The correct answer is A. The combination of a one week incubation period, the Eastern seaboard clue, and the intra-erythrocyte parasites strongly suggest Babesia microti as the infecting organism. The clinical disease is called babesiosis. The infecting protozoan is related to Plasmodium and is transmitted by the bite of the Ixodes dammini tick. Occasional cases have been transmitted by blood transfusions. Most patients are asymptomatic; symptomatic cases in reasonably healthy individuals have the features listed in the question stem. Rare severe cases, which may be fatal, may develop in severely debilitated or asplenic individuals and can be accompanied by severe hemolysis (up to 30% of RBCs may have the parasites) with subsequent hemoglobinuria, hemolysis, and renal failure. Leishmania donovania(choice B) is an intracellular tissue protozoan, not a blood protozoan. Plasmodium falciparum(choice C) and Plasmodium vivax(choice D) are malarial parasites not encountered on the Eastern seaboard. Trypanosoma cruzi(choice E) is an intracellular tissue protozoan, and does not infect blood cells.

A patient is referred to a neurologist because of ataxia. The neurologist diagnoses degeneration of the dorsal columns and dorsal roots of the spinal cord, which has caused impaired proprioception and locomotor ataxia. Which of the following organisms most likely caused this pattern of damage? A. Haemophilus influenzae B. Herpes simplex I C. Neisseria gonorrhoeae D. Neisseria meningitidis E. Treponema pallidum Explanation: The correct answer is E. The findings described are those of tabes dorsalis, a form of tertiary syphilis caused by Treponema pallidum. Tabes dorsalis and other forms of tertiary syphilis are now uncommon in this country, possibly because the common use of antibiotics may "treat" many unsuspected cases of syphilis. Haemophilus influenzae(choice A) and Neisseria meningitidis(choice D) can cause meningitis. Herpes simplex I (choice B) can cause an encephalitis that typically involves the frontal and temporal lobes. Neisseria gonorrhoeae(choice C) causes gonorrhea, which usually does not involve the CNS. Which of the following organisms would be most likely to cause an outbreak of enteritis in a day care center in the United States? A. Helicobacter jejuni B. Salmonella typhi C. Shigella species D. Vibrio cholerae

40 E. Yersinia enterocolitica Explanation: The correct answer is E.Yersinia enterocolitica is an important cause of “mini-epidemics” of pediatric diarrhea. Adults can also be affected, but less commonly than children. Some diarrheal cases are severe (and occasionally fatal) and may be complicated by severe dysentery, appendicitis, or chronic relapsing ileocolitis that may require antibiotics to shorten the course. Helicobacter jejuni(choice A) is an important cause of hospital-acquired diarrhea. Salmonella typhi(choice B) causes typhoid fever. Shigella species (choice C) causes epidemics of dysentery in military camps. Vibrio cholerae(choice D) causes cholera.

A 60-year-old woman has been hospitalized for three weeks with widely metastatic ovarian adenocarcinoma, and she becomes septic with vancomycin-resistant enterococcus. What is the mechanism of vancomycin resistance in this organism? A. Acetylation of antibiotic B. Altered drug-binding protein C. β-lactamase production D. Decreased ribosomal binding of antibiotic E. Formation of novel cell wall peptide bridges Explanation: The correct answer is E. The antibiotic property of vancomycin depends upon its ability to bind D-ala-D-ala, which is vital in the synthesis of peptidoglycan peptide bridges. Vancomycin-resistant enterococci instead utilize D-lactic acid in their peptide bonds, and thus are resistant to the action of vancomycin. Enzymatic deactivation of antibiotics is a common mode of resistance to aminoglycosides, chloramphenicol, and sulfonamides. The most common chemical modifications are acetylation (choice A), adenylation, and phosphorylation. Penicillin that has penetrated the cell membrane is kept in place by binding to penicillin binding proteins (PBPs). Penicillin resistance may be conferred through altered PBP affinity or number (choice B). Vancomycin resistance does not depend on PBPs. β-lactamase (choice C) is a bacterial enzyme that inactivates some of the penicillins and cephalosporins, but has no activity on vancomycin. Modified ribosomes with decreased antibiotic affinity (choice D) confer resistance to protein synthesis-inhibiting antibiotics, including tetracycline, minocycline, doxycycline and the macrolide antibiotics. Vancomycin is not a protein synthesis-inhibiting antibiotic.

41 A 2-year-old boy has surgery to correct a urinary tract obstruction. Post-operatively, with an indwelling urinary catheter in place, he develops a urinary tract infection. Urine culture grows out a lactose-negative, oxidase-positive, gram-negative rod. Which of the following agents is the most likely cause of this boy's urinary tract infection? A. Candida albicans B. Enterococcus faecalis C. Escherichia coli D. Klebsiella pneumoniae E. Proteus mirabilis F. Pseudomonas aeruginosa G. Staphylococcus saprophyticus Explanation: The correct answer is F.Pseudomonas aeruginosa is a gram-negative rod. It can easily be distinguished from the family Enterobacteriaceae because Pseudomonas is oxidase positive. It is an opportunistic pathogen that has an increased chance of causing urinary tract infections in patients with indwelling catheters, or who are on antibiotics. Candida albicans(choice A) is a yeast that can cause urinary tract infections in poorly controlled diabetics, because glucose in the urine enhances its growth. Enterococcus faecalis(choice B) is a gram-positive coccus that commonly causes urinary tract infections in elderly men with prostate problems. Escherichia coli(choice C) is a lactose-positive, oxidase-negative, gram-negative rod that is the most common cause of community acquired urinary tract infections. Klebsiella pneumoniae(choice D) is a lactose-positive, oxidase-negative, gram-negative rod that can cause urinary tract infections in poorly controlled diabetics because glucose in the urine enhances its growth. Proteus mirabilis(choice E) is a gram-negative rod that is a member of family Enterobacteriaceae. It is lactose-negative, but contains urease, which splits urea to yield ammonia, which in turn, raises the pH of the urine, creating a more hospitable environment for the organism. Patients with stones are at increased risk for this organism because it is able to hide in the stones (conversely, Proteus predisposes for the development of stones). Staphylococcus saprophyticus(choice G) is a catalase-positive, coagulase-negative, gram-positive coccus that causes urinary tract infections in young women.

A 65-year-old Laotian immigrant has developed thick, erythematous nodules on her ears and nose with significant associated sensory loss. The nodules have grown slowly over the course of many years. Biopsy of the lesions shows dermal granulomas with giant cells but no acid-fast bacteria. Culture on blood agar and Lowenstein-Jensen medium shows no growth. These findings are most consistent with which of the following diagnoses? A. Cutaneous leishmaniasis

42 B. Onchocerciasis C. Rhinoscleroma D. Smallpox E. Tuberculoid leprosy Explanation: The correct answer is E.Mycobacterium leprae is endemic to parts of Africa, Asia, and South America. Tuberculoid leprosy is an indolent disease, typically affecting cooler parts of the body, such as the nose and ears, producing dermal granulomas with very rare, acid-fast bacilli and damage to peripheral nerves. This is in marked contrast to lepromatous leprosy, which is progressive and invasive and generally characterized by the presence of numerous acid-fast bacteria in a histiocytic, but non-granulomatous tissue response. M. leprae has not been grown in any culture medium. Cutaneous leishmaniasis (choice A) is due to Leishmania species that show a worldwide distribution. The infection is transmitted through a sandfly bite, and the skin lesion is typically ulcerated. Histology shows intracellular parasites within the dermis and epidermis. Granulomas are not formed. Onchocerciasis (choice B), or river blindness, is a roundworm infection transmitted by black flies of Africa and South America. The microfilaria grow at the site of inoculation, and cause an inflamed subcutaneous nodule. The organism is seen on tissue sections. Rhinoscleroma (choice C) is a destructive granulomatous infection of the nasopharynx caused by Klebsiella rhinoscleromatis. Gram-negative rods can be cultured from the lesions. Smallpox (choice D), the infection caused by the variola virus (a DNA poxvirus), produces malaise, headaches, and a macular/pustular rash involving the face and distal extremities. Smallpox has been eradicated through worldwide vaccination.

Which of the following organisms would most likely be isolated from the vagina of a normal 5 year-old girl? A. Candida B. Lactobacillus C. Neisseria D. Staphylococcus E. Streptococcus Explanation: The correct answer is D. The vagina of prepubertal girls and post-menopausal women is colonized by colonic and skin bacteria, including Staphylococcus epidermidis, which is normally found on the skin. The vagina of women of child-bearing age tends to be colonized by Lactobacillus (choice B) species, yeasts such as Candida (choice A), and Streptococcus species (choice E). The presence of Neisseria (choice C), such as N. gonorrhoeae (the cause of gonorrhea), in the vagina of a 5 year-old strongly suggests sexual abuse.

43 A 58-year-old homeless man is brought in to the emergency room by police. His temperature is 101.8F rectally, and he smells of cheap wine. Physical examination is remarkable for dullness to percussion, increased vocal fremitus, and decreased breath sounds and wet crackles on the right. Chest x-ray reveals consolidation of the right upper lobe. One of the nurses notes the production of thick, bloody sputum when the man coughs. The most likely causative organism is A. Haemophilus influenzae B. Klebsiella pneumoniae C. Pseudomonas aeruginosa D. Staphylococcus aureus E. Streptococcus pneumoniae Explanation: The correct answer is B. Although it is far better to rely on microbiology lab results than physical exam and history, some classic clues for an infection with Klebsiella pneumoniae are a patient who is an elderly alcoholic or diabetic, "currant-jelly" sputum (containing blood clots), and lobar pneumonia. Hemophilus influenzae(choice A) is a frequent cause of community-acquired pneumonia, but does not classically produce lobar pneumonia or currant-jelly sputum. Pseudomonas aeruginosa(choice C) classically produces greenish sputum, and is associated with cystic fibrosis rather than alcoholism. Staphylococcus aureus(choice D) often produces pulmonary abscess, but may also produce an ordinary bronchopneumonia. Streptococcus pneumoniae(choice E) is a classic cause of lobar pneumonia, but does not produce currant-jelly sputum or show a particular predilection for alcoholics. An archaeologist who has been excavating a very old Middle Eastern site develops a pustule on his hand. The pustule then ruptures to form a black eschar surrounded by expanding brawny edema. Which of the following organisms is the most likely cause of this condition? A. Bacillus anthracis B. Borrelia burgdorferi C. Francisella tularensis D. Spirillium minus E. Yersinia pestis Explanation: The correct answer is A. Anthrax forms extremely stable spores and has, in fact, been encountered in very old (i.e., deep) dirt in Israeli and Arabian excavation sites. The causative organism, Bacillus anthracis, is found in many animal species, and humans can acquire the organism either through contact with the animals or from locally contaminated soil. The pustule described in the question stem is called a "malignant pustule" and may be accompanied by lymphadenopathy. Most cases remain localized, but death can occur as the result of complications such as bacteremia, meningitis, and pneumonia.

44 Borrelia burgdorferi(choice B) causes Lyme disease. In the first stage of this disease, there is a localized expanding erythematous rash rather than an eschar. Furthermore, Lyme disease occurs in the U.S., Europe, and Asia, corresponding to the distribution of Ixodid ticks, which spread the infection. Francisella tularensis(choice C) causes tularemia. The organism can persist for weeks to months but does not form spores that could survive for hundreds or thousands of years. Also, most human cases occur in the endemic areas of the U.S. Spirillium minus(choice D) is one of the causes of rat-bite fever. There is no indication the man was bitten by a rat. Yersinia pestis(choice E) causes bubonic plague. This organism does not form stable spores that could survive for extended periods of time.

A 33-year-old woman who has recently returned from several years in the Peace Corps in Guatemala presents with severe, acute, right upper quadrant abdominal pain. She recalls that before coming back to the United States, she had several months of bloody diarrhea. CT scan of the liver demonstrates lesions that are interpreted to be abscesses. Which of the following organisms is the most likely cause of her illness? A. Ascaris lumbricoides B. Entamoeba histolytica C. Enterobius vermicularis D. Salmonella typhi E. Shigella species Explanation: The correct answer is B. The patient probably has hepatic amebiasis, which is a life-threatening complication of intestinal infection with Entamoeba histolytica.E. histolytica is transmitted via the fecal-oral route. Intestinal colonization (which may be asymptomatic) always precedes infection of the liver. The abscesses generally contain necrotic debris, with amoebae located along the edges of the abscess. Ascaris lumbricoides(choice A) can cause intestinal obstruction, but does not usually cause hepatic abscesses. Enterobius vermicularis(choice C) is the pinworm, which inhabits the rectum. Bloody diarrhea can also be seen with some strains of Salmonella (choice D) and Shigella (choice E), but these organisms do not usually cause hepatic abscesses. Bacterial causes of hepatic abscesses include E. coli, Klebsiella, Streptococcus, Staphylococcus, Bacteroides, and Pseudomonas.

A previously healthy 11-year-old girl develops a gastrointestinal infection with cramping and watery stools. After several days, she begins to pass blood per rectum, and is hospitalized for dehydration. In the hospital, she is noted to have decreasing urine output with rising blood urea nitrogen (BUN). Total blood count reveals anemia and thrombocytopenia, and the peripheral smear is remarkable for fragmented red cells (schistocytes). Infection with which of the following bacterial genera is most likely responsible for this syndrome? A. Campylobacter

45 B. Clostridium C. Salmonella D. Shigella E. Vibrio Explanation: The correct answer is D. This patient has developed hemolytic-uremic syndrome (HUS), a complication of the Shiga toxin or Shiga-like toxin: exotoxins released by Shigella species and the enterohemorrhagic E.coli. HUS in children usually develops after a gastrointestinal or flu-like illness, and is characterized by bleeding, oliguria, hematuria and microangiopathic hemolytic anemia. Presumably the Shiga toxin is toxic to the microvasculature, producing microthrombi that consume platelets and RBCs, and may fragment the red cell membrane. The incorrect choices are all bacteria which may produce an enterocolitis, but do not elicit HUS. A long-term consequence of Campylobacter (choice A) infection is a reactive arthritis or full-blown Reiter's syndrome. Clostridial enterocolitis is produced by Clostridium difficile(choice B), a normal inhabitant of the gut that produces pseudomembranous colitis when other gut flora are suppressed by treatment with antibiotics. In the United States, Salmonella infections (choice C) are almost all non-typhoid inflammatory diarrhea, producing a simple enterocolitis that may proceed to sepsis in some cases. Typhoid fever (produced by Salmonella typhi and S. paratyphi) produces a protracted illness that progresses over several weeks and includes rash and very high fevers, but not HUS. Vibrio (choice E) infections produce copious amounts of watery diarrhea, and the major risk of cholera and other Vibrio enteritides is shock due to hypovolemia or electrolyte loss.

A 20-year-old man visiting Europe dives into a secluded natural pool. He later develops meningoencephalitis localized to the base of his brain. The causative organism is found to be the free-living amoeba, Naegleria fowleri. Which of the following structures did he probably damage during the dive? A. Cribriform plate B. Lacrimal bone C. Mandible D. Nasal bone E. Zygomatic bone Explanation: The correct answer is A. The rare, but life-threatening, meningoencephalitis caused by Naegleria fowleri apparently ensues when the organism enters the body through the nasal mucosa and crosses the cribriform plate to enter the brain via the olfactory nerves. It has been speculated that trauma to the nose during diving predisposes to this infection. The lacrimal bone (choice B) of the medial aspect of the orbit does not appear to be involved in amoebic infection.

46 The mouth and its associated bones, including the for this organism. mandible (choice C), do not appear to be a route of infection

The nasal bones (choice D), located on the superior surface of the nose, do not appear to be a route of infection for this organism. The zygomatic bone (choice E), which forms the cheekbone, does not appear to be a route of infection for this organism. A 1-year-old girl presents with a 2-day history of fever, vomiting, and watery, nonbloody diarrhea. On physical exam, she appears dehydrated. Which of the following best describes the most likely infecting organism? A. It has a complex double-stranded DNA genome B. It has a partially double-stranded circular DNA genome C. It has a segmented, double-stranded RNA genome D. It has a single-stranded circular RNA genome E. It has a single-stranded RNA genome Explanation: The correct answer is C. Rotavirus is the most common cause of gastroenteritis in children between 3 months and 2 years of age. It is most prevalent in the winter. Rotavirus, one of the reoviruses, looks like a wheel (which ROTAtes) and possesses a double-shelled icosahedral capsid with no envelope. Its genome consists of 11 segments of double-stranded RNA. A complex double-stranded DNA genome (choice A) is found in enteric adenoviruses, the third most common cause of gastroenteritis in infants and children. This organism possesses an icosahedral nucleocapsid. A partially double-stranded circular DNA genome (choice B) is characteristic of hepatitis B. Its envelope contains surface antigen (HBsAg). Its capsid is icosahedral and contains the genome along with DNA-dependent DNA polymerase, which also has reverse transcriptase activity. A single-stranded circular RNA genome (choice D) is characteristic of hepatitis D virus. Its envelope consists of HBsAg. The virus is defective and is able to replicate only in cells infected with hepatitis B. A single-stranded RNA genome (choice E) is characteristic of several viruses that cause gastroenteritis in children, including astrovirus and Norwalk virus. Astrovirus is the second most common cause of viral gastroenteritis in young children. Three structural proteins form its capsid. Norwalk virus is the most common cause of gastroenteritis outbreaks in older children and adults. Its capsid consists of one structural protein.

A 16-year-old male with sickle cell disease is hospitalized for a severe infection. His spleen has autosplenectomized and he has suffered from other minor infections in the past. His symptoms include fever, chills, cough, and chest pain. Bacteria from the patient's sputum yield optochin-sensitive organisms with a positive Quellung reaction. The organism is A. Escherichia coli B. Haemophilus influenzae

47 C. Klebsiella pneumoniae D. Neisseria gonorrhoeae E. Streptococcus pneumoniae Explanation: The correct answer is E. The combination of optochin sensitivity and positive Quellung reaction are properties of a single organism, Streptococcus pneumoniae. The other encapsulated organisms that have Quellung-positive reactions are Haemophilus influenzae(choice B), Neisseria meningitidis, and Klebsiella pneumoniae(choice C). However, none of these organisms are optochin sensitive. The other choices, Escherichia coli(choice A) and Neisseria gonorrhoeae(choice D), are not encapsulated.

Which of the following is specifically associated with infection with Schistosoma haematobium? A. Adenocarcinoma of the bladder B. Adenocarcinoma of the renal pelvis C. Squamous cell carcinoma of the bladder D. Transitional cell carcinoma of the bladder E. Transitional cell carcinoma of the renal pelvis Explanation: The correct answer is C. Carcinomas of the bladder and renal pelvis are usually transitional cell (choices D and E) carcinomas. However, Schistosoma haematobium infection (where Schistosomes lay eggs in the veins near the bladder, thereby inducing a marked inflammatory response) is associated with squamous metaplasia and squamous cell carcinoma of the bladder. Some authors have suggested that medications used to kill the worms may contribute to the etiology. Adenocarcinomas of the renal pelvis and bladder (choices A and B) are rare.

An active intravenous drug abuser presents to the emergency department with fever of 5 days' duration, a cough occasionally productive of blood, and pleuritic chest pain. Petechiae are present in his mouth and conjunctivae, and splinter hemorrhages are visible under the fingernails. Which of the following test results would most likely confirm the identity of the causative agent? A. Antibodies to p24 capsid antigen B. Antibodies to Trichinella spiralis antigen C. Blood culture of a catalase-positive, novobiocin-sensitive, gram-positive coccus D. Blood culture of a coagulase-positive, catalase positive, gram-positive coccus E. Blood culture of a gamma-hemolytic, gram-positive coccus on bile-esculin agar F. Blood culture of an alpha-hemolytic, optochin-resistant, gram-positive coccus

48 G. Blood culture of an alpha-hemolytic, optochin-sensitive, gram-positive coccus Explanation: The correct answer is D. The patient is exhibiting the classic signs of acute bacterial endocarditis. In intravenous drug abusers, Staphylococcus aureus is the most common causative agent, because it is the most common normal flora organism on the skin of these patients, and choice D is the classic description of this species. Catalase positivity distinguishes the genus Staphylococcus from Streptococcus, and Staphylococcus aureus is the only coagulase positive member of its genus. Antibodies to p24 capsid antigen (choice A) would be likely in an HIV-positive drug abuser, rather than a patient with endocarditis. Trichinella spiralis(choice B) can cause splinter hemorrhages during the larval migration period, but would not be the most likely agent given the other symptoms. Staphylococcus epidermidis is a catalase-positive, novobiocin-sensitive, gram-positive coccus (choice C). It is sometimes implicated in subacute bacterial endocarditis, but is less common as a causative agent in drug abusers than in normal individuals. Enterococci are gamma-hemolytic, gram-positive cocci that grow on bile-esculin agar (choice E). Enterococci can cause subacute bacterial endocarditis, chiefly after urologic instrumentation in men, rather than in association with intravenous drug abuse. Streptococcus viridans is an alpha-hemolytic, optochin-resistant, gram-positive coccus (choice F). It is implicated in subacute bacterial endocarditis after oral or dental treatments but would not be the most likely agent, given the acute nature of this patient's presentation. Streptococcus pneumoniae is an alpha-hemolytic, optochin-sensitive, gram-positive coccus (choice G) that produces cough and chest pain, but would be an unlikely cause of bacterial endocarditis.

A 22-year-old woman presents with a 1-week history of mild lower abdominal pain and a yellowish vaginal discharge. She describes the pain as dull in nature, relieved slightly by acetaminophen and worsened by intercourse. Pelvic examination reveals a red, swollen cervix without motion tenderness. The mucosa is friable. Potassium hydroxide (KOH) mount is negative, and wet mount does not reveal clue cells. Gram's stain of the exudate reveals gram-negative cocci. Which of the following procedures would most likely lead to the correct diagnosis? A. Culture the blood on Thayer-Martin agar B. Order DNA probe assays of endocervical exudates C. Order serologic tests to identify specific capsular antigens D. Order the germ tube test E. Order the rapid plasma reagin (RPR) test Explanation: The correct answer is B. The symptoms suggest infection with Neisseria gonorrhoeae; however, with Gram's stain results alone, it is not possible to distinguish the gonococcus from normal flora organisms such as Acinetobacter, unless gram-negative diplococci are found within polymorphonuclear leukocytes. The fastest and most reliable assay specific for diagnosis of gonorrhea is the use of commercial DNA probes, with results available in 2-4 hours.

49 Culture on Thayer-Martin medium (choice A) is indeed the choice for culture of N. gonorrhoeae, but would yield slower results, and therefore be a secondary choice. Serology (choice C) is not a good choice because N. gonorrhoeae does not have a significant capsule (the meningococcus does), and furthermore, serological tests for the gonococcus have proven insensitive and nonspecific. The germ tube test (choice D) is one of the diagnostic tests for Candida albicans, but this case presentation is not characteristic of candidiasis. The RPR (choice E) is a test for reaginic (heterophilic) antibodies formed early in infection with Treponema pallidum, but this case presentation is not suggestive of syphilis. A patient develops fever, shortness of breath, and appears to be quite ill. X-ray demonstrates bilateral interstitial lung infiltrates. Bronchial washings demonstrate small "hat-shape" organisms visible on silver stain within alveoli. Which predisposing condition is most likely to be present in this patient? A. AIDS B. Congestive heart failure C. Pulmonary embolus D. Rheumatoid arthritis E. Systemic lupus erythematosus Explanation: The correct answer is A. The disease is Pneumocystis pneumonia, which is caused by an agent now believed to be a fungus rather than a true bacteria. Pneumocystis carinii pneumonia is seen in immunocompromised patients, particularly in those with AIDS, cancer, and in malnourished children. It can be the AIDS-defining illness. Congestive heart failure (choice B) predisposes the patient to pulmonary edema. Pulmonary embolus (choice C) can cause pulmonary infarction or sudden death. Rheumatoid arthritis (choice D), particularly in miners, can cause formation of lung nodules similar to subcutaneous rheumatoid nodules. Systemic lupus erythematosus (choice E) can cause pleuritis, but is not associated with a significantly increased incidence of pneumonia.

An Hispanic male is referred to the dermatology clinic of a major medical center. On physical examination, the man has several disfiguring lesions on his face and there is loss of cutaneous sensation to fine touch, pain, and temperature. An acid-fast organism is observed in scrapings from a skin lesion. Which of the following organisms is the most likely cause of this patient's disease? A. Bartonella henselae B. Listeria monocytogenes C. Mycobacterium avium-intracellulare

50 D. Mycobacterium leprae E. Nocardia asteroides Explanation: The correct answer is D. The disease in question is leprosy, or Hansen's disease. A key feature in the description is the fact that the organism is acid-fast. Both of the mycobacteria, M. avium-intracellulare and M. leprae are strongly acid-fast, that is they retain the carbol fuchsin dye in the face of acid-alcohol decolorization. M. leprae has a predilection for the skin and cutaneous nerves, thereby producing the symptoms of depigmentation and anesthetic cutaneous lesions. This loss of peripheral nerve function leads to many of the disfiguring features of the disease; because the patients do not have normal pain sensation, they sustain repeated injuries. In addition, the organism attacks cartilage and causes granuloma formation in the skin, leading to some of the facial disfigurement. Bartonella henselae(choice A) is a very small, gram-negative bacterium that is closely related to the rickettsia, although it is able to be cultured on lifeless media. It is the cause of cat-scratch disease, a local, chronic lymphadenitis most commonly seen in children, and bacillary angiomatosis, a disease seen particularly in AIDS patients. Listeria monocytogenes(choice B) is a ubiquitous microbe that causes disease in over 100 animal species. Although it is best known as an agent of meningitis in the newborn, it is a cause of multiple other diseases. A characteristic feature of these infections is the development of granulomas at the site of the infection. The organism is not acid-fast and has no particular predilection for skin or nervous tissues. M. avium-intracellulare(choice C) causes tuberculosis-like pulmonary disease in the immunosuppressed. Nocardia asteroides(choice E) primarily produces pulmonary infections in humans. The organism is consider to be "weakly" acid-fast, meaning that if the amount of HCl used in the decolorization step is reduced, the organisms will retain the carbolfuchsin primary stain.

A 23-year-old man develops explosive watery diarrhea with blood, fecal leukocytes, and mucus approximately 3 days after eating chicken that was improperly cooked. Comma-shaped organisms were found in the fecal smear along with red blood cells and leukocytes. Which of the following pathogens is the most likely cause of these symptoms? A. Campylobacter jejuni B. Enterotoxigenic E. coli C. Shigella sonnei D. Staphylococcus aureus E. Vibrio cholera Explanation: The correct answer is A.Campylobacter jejuni is a pathogen causing an invasive enteric infection associated with ingestion of raw or undercooked food products, or through direct contact with infected animals. In the U.S., ingestion of contaminated poultry that has not been sufficiently cooked is the most common means of acquiring the infection. The patients typically have bloody diarrhea, abdominal pain, and fever. The presence of fecal leukocytes indicates an invasive infection. The organism is a gram negative rod with a "comma-shape."

51 Enterotoxigenic E. coli(choice B) causes the classic traveler's diarrhea. The infection is non-invasive and is acquired via the fecal-oral route through consumption of unbottled water or uncooked vegetables. The major manifestation is a copious outpouring of fluid from the GI tract presenting as explosive diarrhea. This is due to the action of one of two types of enterotoxins on the GI tract mucosa. Shigella sonnei(choice C) produces a syndrome very similar to C. jejuni. However, the organism appears as a gram-negative rod on Gram's stain. It does not have a comma shape. Transmission is from person to person via the fecal-oral route. Infection requires a low infective dose since the organism is fairly resistant to gastric acidity. Staphylococcus aureus(choice D) produces food poisoning due to the ingestion of a pre-formed enterotoxin. The organism is present in food that is high in salt content such as potato salad, custard, milk shakes, and mayonnaise. The patient presents with nausea, vomiting, and abdominal pain, followed by diarrhea beginning 1-6 hours after ingestion of the enterotoxin. Vibrio cholerae(choice E) produces a secretory diarrhea due to increases in cAMP in the intestinal cells. The organism is not invasive. The patient presents with the sudden onset of painless, watery diarrhea that becomes voluminous, followed by vomiting. The stool appears nonbilious, gray, and slightly cloudy with flecks of mucus, no blood, and a sweet odor.

A 37-year-old, intravenous drug-abusing male presents with fever and chills. Blood cultures are positive for Staphylococcus aureus. He develops central nervous system symptoms, and a cerebral abscess is suspected. Which part of the brain is most often affected by septic emboli in patients with infective endocarditis? A. Brainstem B. Cerebellum C. Frontal lobe D. Occipital lobe E. Parietal lobe Explanation: The correct answer is E. Embolization from infective endocarditis typically causes multiple, small parietal lobe abscesses. This "factoid" is worth knowing because some patients with infective endocarditis present with what clinically looks like multiple small "strokes", and their treatable cardiac disease may be completely unsuspected.

A 33-year-old male with AIDS and a history of shingles develops a severe, multifocal encephalitis. Therapy is instituted with acyclovir, but the man dies on the fourth day of his hospital admission. Which of the following viruses is the most likely cause of his encephalitis? A. Cytomegalovirus B. Herpes simplex type I C. Herpes simplex type II D. Herpes zoster-varicella E. Measles virus

52 Explanation: The correct answer is D. The specific clue to the cause of the severe encephalitis in this AIDS patient is the history of shingles, due to reactivation of the herpes zoster-varicella virus. In otherwise healthy adults, the virus (which is usually introduced to the body as a childhood case of chickenpox) remains dormant in a dorsal root ganglion, only to reactivate in later life, causing a painful vesicular eruption that characteristically conforms to the distribution of a single dermatome. In AIDS patients, the virus can cause a severe, multifocal encephalitis that may be resistant to acyclovir therapy. Cytomegalovirus (choice A) can cause disseminated disease (including brain infection) in AIDS patients, but is less likely in this patient, given the past history of shingles. Herpes simplex type I (choice B) and herpes simplex type II (choice C) can cause disseminated disease (including brain infection) in AIDS patients, but is less likely in this patient, given the past history of shingles, and the lack of response to acyclovir. Measles virus (choice E) appears to be related to subacute sclerosing panencephalitis, but this condition is not specifically increased in AIDS patients.

A 35-year-old male undergoes an appendectomy. Several days later, an abscess has formed at the surgical site. It does not improve with administration of a cephalosporin, but does respond to nafcillin. The infecting organism most likely produced an enzyme that would hydrolyze which bond in the above molecule? A. A B. B C. C D. D Explanation: The correct answer is D. Abscesses are often caused by Staphylococcus aureus, an organism that may produce penicillinase, an enzyme that cleaves the amide bond of beta-lactam antibiotics (the molecule shown is penicillin). The enzyme thus confers resistance to the beta-lactam antibiotics (penicillins and cephalosporins). Nafcillin (a semisynthetic penicillin) is very effective against penicillinase-producing S. aureus. You should be familiar with other important enzymes and toxins produced by this organism, including: heat-resistant enterotoxin, toxic shock syndrome toxin, exfoliatin (causes scalded-skin syndrome in children), alpha toxin (kills leukocytes), and coagulase.

53

A 10-year-old boy is attending summer camp in Texas. After 2 weeks of camp, he complains of a sore throat, headache, cough, and malaise. On physical examination, he also has a low-grade fever and keratoconjunctivitis. Within hours, several other campers and counselors visit the infirmary with similar symptoms. All of the patients had been swimming in the camp swimming pool. Eventually, more than 50% of the camp complain of symptoms similar to the initial case that last 5 to 7 days. Which of the following is the most likely causative organism? A. Adenovirus B. Chlamydia C. Gram-negative diplococcus D. Gram-positive enterococcus E. Herpesvirus Explanation: The correct answer is A. Adenoviruses are non-enveloped (naked), icosahedral DNA viruses causing a variety of clinical syndromes. Adenoviruses cause a pharyngoconjunctivitis that affects children and sometimes adults who are living in the same household. Contaminated swimming pools have been implicated as sources for the virus. The virus is latent in the lymphoepithelial tissue of the nasopharynx and other sites. Adenoviruses also cause watery, non-bloody diarrhea. Chlamydia spp. (choice B) produce a variety of clinical syndromes, including a sexually transmitted urethritis, pelvic inflammatory disease, neonatal pneumonia and inclusion conjunctivitis, lymphogranuloma venereum, adult interstitial pneumonia, and a zoonotic pneumonitis. A gram-negative diplococcus (choice C) would be a Neisseria spp. or Moraxella. Neisseria meningitidis would be the logical choice, since it is associated with outbreaks among children. However, the clinical syndrome caused by infection with the meningococcus is one of a fulminating, progressive septicemia and/or meningitis with fever, vascular collapse, and disseminated intravascular coagulation. N. gonorrhoeae does not cause a clinical syndrome as described in the question. Moraxella catarrhalis causes pneumonia in patients with chronic obstructive pulmonary disease. Gram-positive enterococcus (choice D) would refer to Group D streptococcus. These organisms are associated with endocarditis and genitourinary infections. Herpesviruses (choice E) are large, enveloped DNA viruses with an icosahedral shape. Possible diagnoses for this patient include Epstein-Barr virus, producing infectious mononucleosis, or cytomegalovirus, producing a mononucleosis-like syndrome. The patients in the question did not have hepatosplenomegaly (characteristic of mononucleosis), but did have keratoconjunctivitis. The organism causing the outbreak in the question also has a higher level of infectivity and a short incubation time.

A 35-year-old sexually active male presents to his internist with a painless penile vesicle. Physical exam reveals inguinal lymphadenopathy. The infecting organism is definitively diagnosed and is known to exist in distinct extracellular and intracellular forms. Which of the following is the most likely pathogen? A. Calymmatobacterium granulomatis B. Chlamydia trachomatis C. Haemophilus ducreyi

54 D. Neisseria gonorrhoeae E. Treponema pallidum Explanation: The correct answer is B. This patient has lymphogranuloma venereum caused by Chlamydia trachomatis (type L1, 2, or 3). Chlamydia exhibit distinct infectious and reproductive forms. The extracellular infectious form is known as the elementary body (EB), which is incapable of reproduction. It attaches to the host cell and enters through endocytosis. Once inside the cell, the EB is transformed into the reticulate body (RB) within the endosome. The RB is capable of binary fission and divides within the endosome; fusion with other endosomes occurs to form a single large inclusion. Eventually, the RBs undergo DNA condensation and disulfide bond bridgings of the major outer membrane protein, forming EBs. The EBs are then released. Note that C. trachomatis is responsible for several sexually or perinatally transmitted diseases, including ocular trachoma (types A, B, and C), neonatal conjunctivitis, nongonococcal urethritis, cervicitis, and pelvic inflammatory disease (types D-K). Calymmatobacterium granulomatis(choice A) is a gram-negative rod that causes superficially ulcerated genital or inguinal papules that coalesce to form substantial lesions. The appearance of Donovan bodies in histiocytes is diagnostic of this infection. Haemophilus ducreyi(choice C) is a gram-negative rod that causes a soft, painful penile chancre, unlike that of a chlamydial or syphilitic lesion. This infection is common in the tropics. Neisseria gonorrhoeae(choice D) is a gram-negative diplococcus responsible for gonorrhea. Patients typically present with purulent penile discharge, not genital lesions. Treponema pallidum(choice E) is the spirochete responsible for syphilis. It may cause a firm, painless ulcer as a manifestation of primary syphilis, but the organism does not exist in distinct extracellular and intracellular forms as does Chlamydia. Secondary syphilis is associated with the appearance of condyloma lata–flat, gray, wart-like lesions.

A 45-year-old white male with a history of alcohol abuse and periodontal disease is brought to the emergency room for a spiking fever and chills. Physical examination is significant for signs of lung consolidation. A chest x-ray shows a cavity in the right lower lobe that has an air/fluid level. A transtracheal aspiration is performed and the specimen is submitted to the laboratory for routine cultures and Gram's stain. Based upon the clinical presentation, which of the following would be the most likely finding? A. Anaerobic bacteria B. Aspergillus fumigatus C. Entamoeba histolytica D. Staphylococcus aureus E. Streptococcus pyogenes Explanation: The correct answer is A. Anaerobic bacteria derived from the oral flora in the clinical setting of periodontal disease are the most common isolates from lung abscesses. Single lung abscesses are the most common pattern, with the superior segment of a lower lobe or the posterior segment of an upper lobe being affected most often.

55 Aspergillus fumigatus(choice B) presents in the lung as hemorrhagic infarctions, aspergillomas (fungus balls) in cavitary tuberculosis cavities, or as allergic bronchopulmonary aspergillosis. Entamoeba histolytica(choice C) is associated with pulmonary abscesses as an extension of a liver abscess across the diaphragm. Staphylococcus aureus(choice D) usually presents as multiple lung lesions in non-contiguous sites since the spread is embolic. The source of the infection is usually tricuspid endocarditis in IV drug abusers. Streptococcus pyogenes(choice E) typically produces a bronchopneumonia pattern following an upper respiratory infection. A traveler in Bogota, Colombia drinks a glass of fruit juice with ice cubes made from tap water. E. coli contaminating the water supply grow in the traveler's intestine and synthesize a protein that causes his intestinal epithelium to overproduce cyclic AMP, resulting in a watery diarrhea. This syndrome is typical of which of the pathogenic strains of E. coli? A. Enteroaggregative B. Enterohemorrhagic C. Enteroinvasive D. Enteropathogenic E. Enterotoxigenic Explanation: The correct answer is E. Enterotoxigenic E. coli (ETEC), an important cause of traveler's diarrhea, produces a toxin that activates intestinal adenylate or guanylate cyclase. Consequently, the intestinal mucosa overexpresses cAMP, resulting in a mild and self-limited secretory diarrhea. Enteroaggregative E. coli (EAEC; choice A) does not express a toxin, but is seen to cluster over the colonic mucosa in some individuals. Although occasionally found in patients with chronic diarrhea, no clear mechanism for mucosal pathophysiology has been determined for EAEC. Enterohemorrhagic E. coli (EHEC; choice B), classically associated with strain O157:H7 and present in undercooked hamburgers, expresses a Shiga-like toxin that causes bloody diarrhea and hemolytic-uremic syndrome. Enteroinvasive E. coli (EIEC; choice C) does not express a toxin, but rather is pathogenic through its capacity to invade the colonic mucosa and evoke an inflammatory response. The resulting dysentery-like inflammatory diarrhea generally necessitates vigilant hydration, but no antibiotics are indicated. Enteropathogenic E. coli (EPEC; choice D) does not produce any known toxins, but adheres tightly to the glycocalyx of the colonic mucosa and disrupts the microvilli. Villous atrophy, mucosal thinning, and inflammation in the lamina propria, are produced, resulting in impaired absorption and diarrhea.

Zygomycosis, a destructive fungal infection of the sinuses, is likely to reach the brain by which of the following routes? A. Cavernous sinus

56 B. External carotid artery C. Internal carotid artery D. Superior sagittal sinus E. Superior vena cava Explanation: The correct answer is A. The cavernous sinuses are located on either side of the body of the sphenoid bone, and become a potential route of infection because they receive blood both from the face (via the ophthalmic veins and sphenoparietal sinus) and some of the cerebral veins. The spread of infection, especially by Mucor sp., into the cavernous sinus, can produce either CNS infection or cavernous sinus thrombosis, both of which are potentially fatal. The route from the face to the brain is not arterial (choices B and C). The superior sagittal sinus (choice D) is located in the falx cerebri, and drains venous blood from the brain to other dural sinuses, from which it eventually drains into the jugular vein. Zygomycosis does not reach the brain by way of the superior sagittal sinus. The superior vena cava (choice E) drains blood from the upper part of the body into the heart.

A farmer's wife develops abdominal pain and diarrhea, followed several days later by fever, periorbital edema, eosinophilia, and myalgia. She does not remember eating anything unusual recently, but notes that she does make her own pork sausage. Which of the following techniques would be most helpful for the diagnosis of this patient? A. Gastric biopsy B. Muscle biopsy C. Scotch tape test D. Stool for ova E. Stool for protozoal parasites Explanation: The correct answer is B. The patient probably has trichinosis; the diagnosis is confirmed by demonstration of cysts in a muscle biopsy. Trichinosis is uncommon in this country because the organisms are usually found in pork and can be killed by adequate cooking. Cases in the United States are usually due to tasting of raw pork sausage (for seasoning purposes) or ingestion of poorly cooked bear meat. The organism is not usually demonstrated in stool (choices D and E) or gastric biopsies (choice A). The scotch tape test (choice C) is for Enterobius (pinworms).

A 70-year-old man with a history of prostate cancer presents with a chief complaint of pain on the right side of his chest for the past several days. He has been receiving external beam radiation to spinal metastases of his prostate cancer for the past several weeks. On examination, there is marked tenderness along the right side of the chest wall in a 4-6 cm stripe from the midline to the flank. Multiple small vesicular lesions are visible in this

57 area on an erythematous base. Some of the lesions are fluid-filled, and some are crusted. How do members of the virus family responsible for his condition produce messenger RNA? A. By direct translation from the genome B. By producing a double-stranded DNA intermediate C. By producing a negative sense intermediate D. By producing a positive sense intermediate E. By transcribing the genomic DNA F. By transcription from proviral DNA G. The genomic RNA is used directly on the ribosome Explanation: The correct answer is E. This is a classic case of shingles, a stress-activated secondary disease caused by reactivation herpes zoster. Herpesviruses are DNA viruses that use the mechanisms used by our own cells to transcribe an RNA strand from their genomic DNA and use the transcribed RNA as messenger RNA. Positive RNA viruses use direct translation from the genome (choice A) to make protein, not messenger RNA. Retroviruses produce a double-stranded DNA intermediate (choice B) to effect genomic duplication. Positive sense RNA viruses produce a negative sense intermediate (choice C) to produce a genomic duplicate. Negative sense RNA viruses produce a positive sense intermediate (choice D) to produce messenger RNA and a genomic duplicate. Transcription from proviral DNA (choice F) is used by the retroviruses to make messenger RNA. Genomic RNA used directly on the ribosomes (choice G) characterizes the production of messenger RNA by positive sense RNA viruses. A 66-year-old man with urinary retention secondary to prostatic hyperplasia develops a spiking fever and tachypnea. Physical exam reveals intercostal muscle retractions and bilateral inspiratory crackles. A chest x-ray exhibits bilateral interstitial and alveolar infiltrates. Arterial blood gases demonstrate severe hypoxemia. Blood cultures would most likely reveal A. gram-negative diplococci B. gram-negative rods C. gram-positive cocci D. gram-positive diplococci E. gram-positive rods Explanation: The correct answer is B.Escherichia coli is a very common pathogen associated with urinary tract infections and is a common cause of cystitis. It is part of the normal flora of the GI tract. Patients with cystitis can develop

58 bacteremia and subsequent septic shock and adult respiratory distress syndrome (ARDS). The patient described in the history has an obstructive lesion of the urinary tract that predisposes him to the overgrowth of microorganisms such as E. coli. Pulmonary symptoms are consistent with ARDS. E. coli is a gram-negative rod. Gram-negative diplococci (choice A) might be Neisseria spp. or Moraxella catarrhalis. Neisseria gonorrhea is the cause of gonorrhea, a sexually transmitted disease that presents with urethritis, or may be asymptomatic. Patients are usually younger, sexually active males. Moraxella spp. and Kingella kingae are gram-negative cocci that can cause a wide variety of infections. Moraxella catarrhalis is usually implicated as a cause of otitis media and sinusitis in children, or as a cause of purulent tracheobronchitis and pneumonia in a population of patients who are over 50 and have underlying obstructive lung disease. Gram-positive cocci (choice C), such as Staphylococcus aureus and S. epidermidis and Streptococci, rarely cause cystitis. Staphylococcus saprophyticus causes urinary tract infections, but the patients are typically young, sexually active women. Gram-positive diplococci (choice D) would be a description of Streptococcus pneumoniae, which is the most common cause of community-acquired pneumonia. Gram-positive rods (choice E) would include members of the following genera: Clostridium, Bacillus, Listeria, and the coryneform bacteria. The only significant member of this group to produce urinary tract infections is Corynebacterium urealyticum. (C. jeikeium). The organism creates an alkaline urine environment with the potential for stone formation. Patients are usually immunocompromised or have had recurrent urinary tract infections.

A sexually active 25-year-old man develops epididymitis and orchitis. Needle biopsy demonstrates a prominent leukocytic infiltrate with numerous neutrophils. Which of the following organisms is the most likely cause of this man's infection? A. Escherichia coli B. Mycobacterium tuberculosis C. Neisseria gonorrhoeae D. Pseudomonas sp. E. Treponema pallidum Explanation: The correct answer is C. Acute epididymitis and orchitis with prominent neutrophils in a sexually active male are most likely due to infection with Neisseria gonorrhoeae or Chlamydia trachomatis. N. gonorrhoeae can produce a nonspecific pattern of acute inflammation (nonspecific epididymitis and orchitis) or can be sufficiently severe as to cause frank abscesses within the epididymis. Escherichia coli(choice A) is an important cause of nonspecific epididymitis and orchitis in children with congenital genitourinary abnormalities and in older men. Mycobacterium tuberculosis(choice B) can cause tuberculosis of the epididymis and testes, characterized by granuloma formation. Pseudomonas sp.(choice D) has been implicated as an important cause of nonspecific epididymitis and orchitis in older men. Treponema pallidum(choice E), the causative agent of syphilis, can cause testicular involvement with gumma formation, endarteritis, and/or a prominent plasma cell infiltrate.

59 Which of the following organisms is the most common cause of community-acquired pneumonia? A. Chlamydia pneumoniae B. Haemophilus influenzae C. Mycoplasma pneumoniae D. Staphylococcus aureus E. Streptococcus pneumoniae Explanation: The correct answer is E. The most common bacteria implicated in community-acquired pneumonia is the pneumococcus, Streptococcus pneumoniae. Other organisms frequently implicated in patients less than age 60 without comorbidity include Mycoplasma pneumoniae, respiratory viruses, Chlamydia pneumoniae, and Haemophilus influenzae. When community-acquired pneumonia occurs in elderly patients or patients with comorbidity, aerobic gram-negative bacilli and Staphylococcus aureus are added to the list. The organisms listed in choices A, B, and C are important causes of community-acquired pneumonia, but are not the most frequent causes. Staphylococcus aureus(choice D) is an important cause of community-acquired pneumonia (particularly in the elderly and in patients with comorbidity), but is not the most frequent cause.

A patient with a history of chronic diarrhea and bloody stools presents to the emergency department with right upper quadrant pain and fever. Physical examination demonstrates a large, tender liver. Which of the following would be most likely to have caused the patient's problems? A. Cryptosporidium parvum B. Entamoeba histolytica C. Giardia lamblia D. Isospora belli E. Trichomonas vaginalis Explanation: The correct answer is B.Entamoeba histolytica is an intestinal amoebic parasite that either can cause relatively mild diarrhea, or can behave more aggressively, causing dysentery (bloody diarrhea with abdominal pain and dehydration), peritonitis, or liver abscess formation (such as this patient has). Treatment is with metronidazole. Cryptosporidium parvum(choice A), Giardia lamblia(choice C), and Isospora belli(choice D) can cause chronic diarrhea, but would not be expected to cause a liver abscess. Trichomonas vaginalis(choice E) is not a cause of diarrhea, but instead causes vaginitis.

60 A neonate is born in very poor condition, with a severe, generalized encephalitis. Which of the following viruses is the most likely pathogen in this setting? A. Eastern equine encephalitis virus B. Herpes simplex type II C. Herpes zoster-varicella virus D. Poliomyelitis virus E. St. Louis encephalitis virus Explanation: The correct answer is B. Viral causes of neonatal encephalitis include three members of the herpes family of viruses: herpes simplex I, herpes simplex II, and cytomegalovirus. All three types can have devastating effects on the neonate, with extensive CNS damage leading to mental retardation, seizures, and focal neurologic problems. Acyclovir may be of some help in modifying these infections, but both treatment and prognosis remain very problematic. Eastern equine encephalitis virus (choice A) and St. Louis encephalitis virus (choice E) are causes of epidemic encephalitis but are not the most likely cause of neonatal encephalitis. Herpes zoster-varicella virus (choice C), unlike herpes simplex, is not usually a cause of neonatal encephalitis. Poliomyelitis virus (choice D) is a gastrointestinally transmitted virus that is not usually encountered in neonates (or anyone else in the U.S. currently).

A 15-year-old high school student and several of her friends ate lunch at a local Chinese restaurant. They all were served the daily luncheon special, which consisted of sweet and sour pork with vegetables and fried rice. All of the girls developed nausea, vomiting, abdominal pain, and diarrhea within 6 hours of eating lunch. Which of the following is the most likely cause of these symptoms? A. Bacillus cereus B. Clostridium botulinum C. Clostridium perfringens D. EHEC (Enterohemorrhagic E. coli) E. Staphylococcus aureus F. Vibrio cholerae Explanation: The correct answer is A.Bacillus cereus produces a self-limited diarrhea due to ingestion of the preformed enterotoxin in contaminated fried rice and seafood. The incubation period is typically around 4 hours. The degree of vomiting is greater than the diarrhea. B. cereus is also associated with keratitis, producing a corneal ring abscess. Clostridium botulinum(choice B) produces a neurotoxin that blocks the release of acetylcholine, resulting in a

61 symmetric descending paralysis that may lead to respiratory complications causing death. Symptoms include blurred vision, photophobia, dysphagia, nausea, vomiting, and dysphonia. Most cases are associated with the ingestion of contaminated home-canned food. Clostridium perfringens(choice C) produces a severe diarrhea with abdominal pain and cramping (sometimes called "church picnic" diarrhea). The incubation period is 8-24 hours after ingesting contaminated meat, meat products, or poultry. The meats have usually been cooked, allowed to cool, and then warmed, which causes germination of the clostridial spores. EHEC-Enterohemorrhagic E. coli(choice D), produces a bloody, non-invasive diarrhea due to the ingestion of verotoxin found in undercooked hamburger at fast food restaurants. The 0157:H7 serotype typically produces this syndrome. Some patients develop a life-threatening complication called hemolytic-uremic syndrome. Staphylococcus aureus(choice E) produces a self-limited food poisoning syndrome with nausea, vomiting, and abdominal pain followed by diarrhea beginning 1-6 hours after ingestion of the enterotoxin. The organism is found in foods such as potato salad, custard, milk shakes, and mayonnaise. Vibrio cholerae(choice F) typically produces a watery, non-bloody diarrhea with flecks of mucus (rice-water stools). Abdominal pain is not a feature. Massive fluid loss and electrolyte imbalance are complications. In the U.S., cases of cholera (El Tor 01 strain) are associated with the Gulf coast and ingestion of poorly cooked or poorly stored crabs, shrimp, or oysters. A strain of V. cholerae called non-01 is also found along the Gulf coast. Patients who ingest contaminated shellfish experience fever, copious watery diarrhea, and abdominal cramps within 48 hours after eating. A 29-year-old news correspondent returns from covering an earthquake and its aftermath in a third world country. The man feels tired and has sore muscles, so he consults a physician, who examines him, but decides not to admit him to the hospital. The man subsequently develops a disabling illness complicated by severe anemia, pulmonary edema, renal failure, and shock. Which of the following organisms is the most likely pathogen? A. Babesia microti B. Plasmodium falciparum C. Plasmodium malariae D. Plasmodium ovale E. Plasmodium vivax Explanation: The correct answer is B. Most forms of malaria cause chronic disease with significant morbidity but very little mortality. The exception is malaria caused by Plasmodium falciparum (sometimes still called malignant tertian malaria), which has a propensity for being severe and having life-threatening complications. Falciparum malaria may have very severe anemia with hemoglobin less than 5 g/dL and more than half of the erythrocytes bearing parasites. Complications of this form of malaria include pulmonary edema, renal failure with or without hemoglobinuria, shock, hypoglycemia, and cerebral malaria. Episodes occur with an irregular periodicity with fever spikes at 48-hour intervals during symptomatic periods. Babesia microti(choice A) is an intraerythrocytic parasite that causes relatively mild, self-limited disease called babesiosis. Plasmodium malariae(choice C) causes relatively mild malaria characterized by fever spikes with a 72-hour periodicity. Plasmodium ovale(choice D) and Plasmodium vivax(choice E) cause relatively mild malaria characterized by fever spikes with a 48-hour periodicity.

62 A 9-month-old infant is brought to the Health Department to receive the second dose of OPV (oral polio vaccine) 2 weeks after the first vaccination. The child has mild diarrhea, so the decision is made to defer further immunizations. Bacteriologic examination of a stool culture is unremarkable; however, a small, single-stranded, positive RNA virus is isolated from the specimen. This same agent was isolated from sewage effluent the preceding week. The viral isolate was not inactivated by ether. Which of the following viruses was most likely isolated? A. Adenovirus B. Hepatitis C C. Parvovirus B19 D. Poliovirus E. Rotavirus Explanation: The correct answer is D. Poliovirus, which is a single-stranded +RNA virus, is naked (i.e., non-enveloped) and hence will not be inactivated by lipid solvents such as ether. The live virus vaccine had colonized the intestinal tract of the infant and was still being shed 2 weeks after the earlier oral dose. This same virus, the vaccine strain, is likely to be found in sewage, as all vaccinated infants will shed virus for a period of time after immunization with OPV. Adenoviruses (choice A) and parvovirus B19 (choice C) also may cause diarrheal disease and both are non-enveloped; however, they both have a DNA genome. Hepatitis C (choice B) is an enveloped, single-stranded +RNA virus; its major target organ is the liver, not the intestinal tract. It is a fragile agent that does not survive well outside the body and would not be isolated from raw sewage effluent. Rotavirus (choice E) is the major cause of diarrheal disease in infants under the age of 2 years. It is a member of the reovirus family and, as such, is double-stranded. This virus causes hospitalization of 30% to 40% of the infected infants and kills hundreds of thousands of infants in developing nations where access to hospitals is not readily available. Therapy for the watery diarrhea produced by this agent is fluid and electrolyte replacement. A 24-year-old male Asian immigrant presents with an ulcerative genital lesion. The lesion first appeared 1 month ago as a papule with an erythematous base, which eventually became ulcerated and painful. On physical examination, the man is afebrile. A tender ulcerative lesion is present on his prepuce, and inguinal adenopathy is evident. Which of the following would be the most likely microscopic finding in a scraping from the rash? A. Epithelial cells with intranuclear inclusion bodies B. Iodine-staining intracellular inclusion bodies C. Koilocytotic squamous epithelial cells D. Neutrophils containing gram-negative diplococci E. Pleomorphic gram-negative rods in a "school of fish" pattern F. Spirochetes visible by darkfield microscopy

63 Explanation: The correct answer is E. This is a typical case description of chancroid, caused by Haemophilus ducreyi, a pleomorphic gram-negative rod that displays a characteristic pattern on Gram's stained slides. Epithelial cells with intranuclear inclusion bodies (choice A) would be found with herpes simplex infections, but these lesions would not have the appearance described in this case history. Iodine-staining intracellular inclusion bodies (choice B) would be found with genital lesions of Chlamydia trachomatis, the causative agent of lymphogranuloma venereum, but this lesion is generally nonpainful. Koilocytotic squamous epithelial cells (choice C) would be found in infections with human papilloma virus (HPV), which is associated with venereal warts. Neutrophils containing gram-negative diplococci (choice D) would be found if this were gonorrhea, but the case symptoms are not consistent with this disease. Spirochetes (choice F) would be found on darkfield microscopy if this were a case of syphilitic chancre, but that chancre would be hard and nontender.

A burn patient at the university hospital has been progressively deteriorating. He was catheterized for several days and developed a severe pneumonia, for which he was intubated and is now ventilator-dependent. A gram-negative, non-fermenting rod is isolated from his sputum. It produces a blue-green pigment on growth media and has a grape-like fruity odor. The organism most likely isolated is A. Escherichia coli B. Klebsiella pneumoniae C. Legionella pneumophila D. Pseudomonas aeruginosa E. Serratia marcescens Explanation: The correct answer is D. Pseudomonas aeruginosa is a very common opportunist in burn patients, in whom it classically causes secondary wound infections and septicemia. It may also cause cystitis in patients with urinary catheters and pneumonia in patients with cystic fibrosis. The organism is found in water and usually gains access to the body as a contaminant in the water used in respirators or in water baths used to cleanse wounds. This organism is a non-fermenter, that is, it does not metabolize sugars by classic pathways. It produces a blue-green, water-soluble pigment (pyocyanin), and has a fruity odor when grown on laboratory media. Escherichia coli(choice A) is a lactose-fermenting, gram-negative rod commonly seen as normal flora of the intestine of man. It is the most common cause of urinary bladder infections, pyelonephritis, and sepsis in patients with indwelling urinary catheters. It is also the major cause of traveler's diarrhea and is a very important pathogen in neonates who become infected during passage through the birth canal. Klebsiella pneumoniae(choice B) is a gram-negative, highly encapsulated rod that is a significant pulmonary pathogen in individuals with a respiratory compromise. It is a common cause of aspiration pneumonia and pulmonary abscesses in alcoholics and patients with chronic obstructive pulmonary disease. The organism is readily grown on standard laboratory media such as blood agar or MacConkey's enteric agar.

64 Legionella pneumophila(choice C) is a fastidious, gram- negative respiratory pathogen that may cause either a fulminating disease or a mild "walking pneumonia-like" condition (i.e., an atypical pneumonia). The organism can be cultured on a charcoal yeast extract medium, but identification is usually accomplished by immunofluorescent examination of the pulmonary specimen. Serratia marcescens(choice E) is a gram-negative organism that is found in water, soil, and as an occasional normal flora of humans. It is an opportunistic pathogen that causes respiratory disease in hospitalized patients. Many strains produce a pigment, but the colonies are usually red, pink, or orange. Interestingly, the pathogenic varieties are most often non-pigmented.

A 57-year-old fisherman with a history of alcoholism is hospitalized in Gulfport, Mississippi with a 1-day history of severe, watery diarrhea after eating several raw oysters. He is badly dehydrated on admission, and within 12 hours, he becomes severely hypotensive and dies. Which of the following pathogens is the most likely cause of this man's death? A. Citrobacter diversus B. Enterotoxigenic E. coli C. Providencia stuartii D. Vibrio cholerae E. Vibrio vulnificus Explanation: The correct answer is E.Vibrio vulnificus is an extremely invasive organism, producing a septicemia in patients after eating raw shellfish, or causing wound infections, cellulitis, fasciitis, and myositis after exposure to seawater or after cleaning shellfish. Patients at high risk for septicemia include those with liver disease, congestive heart failure, diabetes mellitus, renal failure, hemochromatosis, and immunosuppression. Citrobacter diversus(choice A) produces neonatal meningitis and can be frequently cultured from the umbilicus. Enterotoxigenic E. coli(choice B) produces the classic traveler's diarrhea. The toxin is ingested in water and salads. The incubation period is approximately 12 hours. The diarrhea is non-inflammatory and treatment is supportive. Providencia stuartii(choice C) is a gram-negative rod related to Proteus. It is a common cause of nosocomial bacteremia in nursing home patients with chronic catheterization. Vibrio cholerae(choice D) produces a non-invasive, non-inflammatory, high-volume secretory diarrhea that is toxin-mediated. A 38-year-old AIDS patient presents to his physician's office in Kansas City, Missouri, complaining of fever for the past week and an increasing headache. He also states that sunlight hurts his eyes and that he has been feeling nauseated and weak. His past medical history is significant for Pneumocystis pneumonia and a total CD4 count of 89. Current medications are trimethoprim/sulfa and indinavir. Cerebrospinal fluid (CSF) reveals 4 WBC, and budding encapsulated yeast forms grow on Sabouraud's agar. Which of the following is an accurate description of the morphology of the infectious form of the organism responsible for the man's illness? A. Broad-based, budding yeasts B. Budding yeasts in a "pilot's wheel" arrangement

65 C. Cylindrical arthroconidia D. Encapsulated budding yeasts E. Filamentous molds F. Septate hyphae with microconidia and macroconidia Explanation: The correct answer is D. This is a classic case of Cryptococcus neoformans meningitis. Clues included the patient population (HIV positive), geographic area (Mississippi and Missouri river beds), and diagnostic form (encapsulated yeast). Cryptococcus is a monomorphic fungus, unlike many of the classic pathogens within the fungal group, so the encapsulated yeast form would be found both in clinical specimens and in the environment as the infectious form. Broad-based, budding yeasts (choice A) would be the forms expected to be found in clinical specimens (not environmental forms) from patients infected with Blastomyces dermatitidis, which is far more likely to present with skin and bone lesions than with meningitis. Budding yeasts in a "pilot's wheel" arrangement (choice B) would be the form expected to be found in clinical specimens (not environmental forms) from patients infected with Paracoccidioides brasiliensis, which is limited in geographic region to Central and South America and typically presents as a primary pulmonary disease. Cylindrical arthroconidia (choice C) would be the transmission forms characteristic of Coccidioides immitis, which may be a cause of fungal meningitis in immunologically compromised individuals, but is geographically restricted to the sub-Sonoran desert zone of the U.S. (San Joaquin valley fever). Filamentous molds (choice E) are the transmission forms of several of the fungal agents (eg, Coccidioides, Blastomyces, Paracoccidioides) but Cryptococcus does not have a filamentous form. Septate hyphae with microconidia and macroconidia (choice F) are the transmission (environmental) forms of Histoplasma capsulatum, which is not an encapsulated yeast in spite of its name. It is primarily a pulmonary infection acquired by exposure to the droppings of birds or bats.

S-ar putea să vă placă și